SlideShare a Scribd company logo
RO Full Length
Paper#1
Target PCS Lucknow
https://targetpcslucknow.com/
Whatsapp/Call @ 7390023092
RO Full Length Paper#1
Target PCS Lucknow Page 1
Time Allowed: 2 Hours Maximum Marks: 140
INSTRUCTIONS
1. IMMEDITELY AFTER THE COMMENCEMENT OF THE EXAMINATION, YOU SHOULD
CHECK THAT THIS TEST BOOKLET DOES NOT HAVE ANY UNPRINTED OR TORN OR MISSING PAGES OR
ITEMS, ETC. IF SO, GET IT REPLACED BY A COMPLETE TEST BOOKLET.
2. You have to enter your Roll Number on the Test
Booklet in the Box provided alongside. DO NOT
Write anything else on the Test Booklet.
4. This Test Booklet contains 150 items (questions). Each item is printed only in English. Each item
comprises four responses (answers). You will select the response which you want to mark on the
Answer Sheet. In case you feel that there is more than one correct response, mark the response
which you consider the best. In any case, choose ONLY ONE response for each item.
5. You have to mark all your responses ONLY on the separate Answer Sheet provided. See directions
in the Answer Sheet.
6. All items carry equal marks.
7. Before you proceed to mark in the Answer Sheet the response to various items in the Test
Booklet, you have to fill in some particulars in the Answer Sheet as per instructions sent to you with
your Admission Certificate.
8. After you have completed filling in all your responses on the Answer Sheet and the examination
has concluded, you should hand over to the Invigilator only the Answer Sheet. You are permitted to
take away with you the Test Booklet.
9. Sheets for rough work are appended in the Test Booklet at the end.
10. Penalty for wrong answers:
THERE WILL BE PENALTY FOR WRONG ANSWERS MARKED BY A CANDIDATE IN THE OBJECTIVE TYPE
QUESTION PAPERS.
(i) There are four alternatives for the answer to every question. For each question for which a wrong
answer has been given by the candidate, one-third of the marks assigned to that question will be
deducted as penalty.
(ii) If a candidate gives more than one answer, it will be treated as a wrong answer even if one of the
given answers happens to be correct and there will be same penalty as above to that question.
(iii) If a question is left blank, i.e., no answer is given by the candidate, there will be no penalty for
that question.
RO Full Length Paper#1
Target PCS Lucknow Page 2
1.consider the following pillers:
Revolutionary Organisation Related Person
(1) Anushilan samiti Promotha mitter
(2) Ramosi Peasant Force Bhagwan singh
(3) Mitra mela V.D.Sawarkar
(4) Anjuman-i-Mohisban-i-Watan Ajit Singh
Which of the pairs given above are correctly matched ?
(a) 1 and 2 only
(b) 3 and 4 only
(c) 1,3 and 4 only
(d)1, 2,3 and 4 only
Explanation: C
Pairs 1,3 and 4 are correctly matched. Balwant Fadke is related to Ramosi Peasant Force
2. In the context of cultural history of India, the terms ghatikas and matha refers to:
(a) traditional pottery works
(b) educational institutions
(c) poetic literary compositions
(d) commercial centers of trade and Commerce
Explanation: B
Terms ghatikas and matha refers to educational institutions
3.which of the following book were not written by Mahatma Gandhi :
1.Hind swaraj
2. Village Swaraj
3.A study in karma
4.Constructive programme –its meaning and places
Explanation: C
RO Full Length Paper#1
Target PCS Lucknow Page 3
“A study in karma” is written by Annie Besant
4. The European planters had been forcing the peasant to grow indigo on ____part of the
total land
1. 3/10
2. 3/20
3. 3/30
4. 3/50
Explanation: B
5. The theme of Ravana Shaking mount kailash is found at
(a) Ajanta Caves
(b) Ellora Caves
(c) Barabar Caves
(d) Bhimbetka Caves
Explanation: B
The theme of Ravana shaking Mount Kailasha has been found in the caves of Ellora. The
image is dated to the eighth century CE.
6. Kiratarjuniyam is a composition of which among the following poets?
(a) Kalidas
(b) Dandin
(c) Bharvi
(d) None of the above
Explanation: C
While some believe that it is the story of the descent of the Ganga from heaven to earth,
others believe that the main story is of Kiratarjuniya or Arjuna‘s penance, a poetic work by”
Bharvi “ which is known to have been popular in the Pallava court.
7. The term, “sulh-i kul” means:
(a) The religion of God
(b) Universal peace
RO Full Length Paper#1
Target PCS Lucknow Page 4
(c) The rules of war
(d) None of the above
Explanation: B
Sulh-i kul means ―universal peace‖
8. In Buddhism “Upasaka” means
a. Lay Worshipper
b. Death of Buddha
c. Tantric Buddhism
d. First Sermon
Explanation: B
9. The Nalayira Divya Prabandham was composed by the:
(a) The Alvars
(b) The Nayanars
(c) The Lingayats
(d) None of the above
Explanation: B
The Nalayira Divya Prabandham is one of the major anthologies of compositions by the
Alvars.It was frequently described as the Tamil Veda
10. The Razmnama is a translated version of Indian epic:
(a) Mahabharata
(b) Ramayana
(c) Upanishads
(d) None of the above
Explanation: A
Translations of Sanskrit texts such as the Mahabharata and the Ramayana into Persian
were commissioned by the Mughal emperors. The Mahabharata was translated as the
Razmnama (Book of Wars)
11. The chief items of export in the Vijayanagar Empire were
RO Full Length Paper#1
Target PCS Lucknow Page 5
(a) Horses, pearls and rice
(b) Cotton, iron and saltpetre
(c) Saltpetre, spices and horse
(d) Copper, silk and spices
Explanation: B
According to the accounts of the foreign travellers, the Vijayanagar Empire was one of the
wealthiest parts of the world at that time. The chief items of exports were cotton and silk
clothes, spices, rice, iron,saltpetre and sugar. The imports consisted of horses, pearls,
copper, coral, mercury, China silk and velvet clothes.
12. Consider the following pairs is not correct
Leaders Area of Revolt, 1857
a. General Bakht Khan Bareilly
b. Nana Sahib Poona
c. Hazrat Mahal Lucknow
Explanation: B
At Kanpur, the Revolt was led by Nana Sahib, the adopted son of Peshwa Baji Rao II. The
most loyal servants of Nana Sahib were Tantia Tope and Azimullah.
13. Tirath Singh is relating to which revolt
a. Kandh
b. Santhal
c. Khasi
d. Kol
Explanation: C
The Khasis, Garos, Khamptis and the Singhpos organised themselves under Tirath Singh to
drive away the strangers from the plains. The uprising developed into a popular revolt
against the British rule in the area. By 1833, the superior English military force had
suppressed the revolt.
14. The Jain Philosophy holds that the world is created and maintained by
(a) Universal Soul
RO Full Length Paper#1
Target PCS Lucknow Page 6
(b) Universal Truth
(c) Universal Faith
(d) Universal Law
Explanation: D
Like the Charvakas, the Jains too do not believe in the Vedas, but they admit the existence
of a soul. They believed that the world is created and maintained by Universal Law.
15. The outcome of “Shimla Deputation “was
a.Government of india act of 1919
b. Indian council act of 1909
c.Indian council act of 1892
d.Government of india act of 1935
Explanation: B
In October 1906, a group of muslim elties called the shimla deputation ,led by the Aga khan ,
met lord minto and demanded separate electorates for the muslims. with reference to these
demands Indian council act of 1909 was passed .
16.Who is called the father of india archeology
a.Mortimer wheeler
b.John marshall
c.James prinsep
d.Alexander Cunningham
Explanation: D
17. Which of the following statements regarding Tropical cyclone is incorrect ?
(a) These are developed only on the hot ocean surface
(b) These are formed by different fronts
(c) Every parts of tropical cyclone gets rain
(d) These are not originated at equator.
Explanation: B
Temperate cyclone are formed by fronts not tropical cyclone.
18. "Kanto" is the largest plain of which country?
(a) Itly
(b) Mexico
RO Full Length Paper#1
Target PCS Lucknow Page 7
(c) Japan
(d) China
Explanation: C
Kanto is the largest plain of japan . This is the geographical region of the biggest island of
japan "Honshu"
19. Sargasso Sea is part of which ocean?
(a) Arctic ocean
(b) Atlantic ocean
(c) Pacific ocean
(d) Indian ocean
Explanation: B
The Sargasso Sea is part of the Atlantic Ocean or the center of marine flora called "Plantak"
produced by mixing of hot and cold water streams where fish production is high.
20.The "Kainchee Dam" is built on which river?
(a) Niger
(b) Nile
(c) Zambeji
(d) Amazon
Explanation: A
Kainchee Dam is located on Niger river.Sennar Dam is located on Blue Nile and Kariba Dam
is located on Zambezi River.
21.Which of the following areas in the world is not basically dependent on agriculture?
(a) The Mediterranean Sea coastal region
(b) Egypt's Nile Valley
(c) Brahmaputra Valley
(d) North-East USA.
Explanation: D
The North Eastern region of the USA is known as the Developed Industrial Area. Major
industrial cities here are Bostow , Portland,
Road Iceland Province, Massachusetts etc. Industrial cities which are not famous for agro
based industry sector. The rest of the above options basically depend on agriculture.
22.Which of the following gulfs has the world's highest tide?
(a) Bay of Bengal
(b) Bay of Hudsan
(c) Bay of Fundy
(d) Bay of Khambhat
Explanation: C
The highest tide in the world comes in the Bay of Fundy, located on the eastern border of
USA and Canada, which is 20% higher than the other Tides.
23.In which state of India is the Subansiri Hydroelectric Project located?
(a) Manipur
RO Full Length Paper#1
Target PCS Lucknow Page 8
(b) Meghalaya
(c) Arunachal pradesh
(d) Tamilanadu
Explanation: C
Subansiri Hydroelectric Project is located on the Subansiri River in Arunachal Pradesh.
24.At what altitude is the alpine vegetation group found above average sea level?
(a) 2200-3200
(b) 3200-3700
(c) 3200-4200
(d) 4200-5200
Explanation: B
Alpine flora has an average elevation of 32 to 37 meters above sea level. Major trees include
silver,fur,juniper, etc. These are economically important.
25.Correct ascending order of creation of following states-
(a) Nagaland, Meghalaya, Sikkim, Arunachal Pradesh
(b) Meghalaya, Arunachal Pradesh , Nagaland, Sikkim
(c) Arunachal pradesh, Nagaland,Sikkim, Meghalaya
(d) Sikkim, Nagaland, Arunachal pradesh, Meghalaya
Explanation: A
Nagaland was formed in 1963, Meghalaya was formed in 1972, Sikkim was formed in 1975
and Arunachal Pradesh was formed in 1987.
26.From which of the following places copper is mined?
(a) Kudremukh
(b) Koraput
(c) Singreni
(d) Khetri
Explanation: D
Copper is produced from Khetri Mining in Rajasthan, while iron ore is produced from
Kudremukh and coal is produced from Singareni.
27.Which of the following is a radio wave reflected from atmospheric layers, which is
transmitted from the Earth and then back to the Earth?
(a) Mesosphere
(b) Ionosphere
(c) Stratosphere
(d) Troposphere
Explanation: B
Radio waves sent from the Earth to the ionosphere at an altitude of 90 km are reverted so
that we can hear or watch broadcasts on our electronic devices.
28.Which of the following statements is not correct with reference to India
(a) Iron is abundant in red soil.
(b) Phosphorus, nitrogen and organic matter are found in abundance in black soil.
(c) Alluvial soil contains a significant amount of potash, but it contains a very small
amount of phosphorus.
RO Full Length Paper#1
Target PCS Lucknow Page 9
(d) Red soil is suitable for the production of brides and coarse grains.
Explanation: B
Black soil is formed by the cooling of lava material from volcanic eruptions. This soil is rich
in iron, aluminum, magnesium and lime and nitrogen phosphorus and organic matter are not
found. Also called regur soil or wheat is world famous for cotton production.
29.Which of the following is not correct?
(a) Lake Titicaca -Bolivia - Peru
(b) Urmila Lake- Iran
(c) Reindeer Lake- Russia
(d) Athabasca Lake -Canada.
Explanation: C
Reindeer Lake is located in Canada. Its depth is 337 meters. Winnipeg Great Bear Great
Slave heads in other Canadian lakes.
30.Which of the following statements is not correct?
(a) The Boss Strait is located between Australia and Tasmania.
(b) The Terrace Strait is located between Australia and New Guinea.
(c) The Cook Strait separates the 2 big countries of New Zealand.
(d) The Great Australian Bay lies to the east of Australia
Explanation: D
The Great Australia Bay is an open bay with a coastline in the central western part of
Australia. All other statements are true.
31.Which one of the following is not correctly matched?
Tribes. Places
(a) Kol. Rewa
(b) Mudiya. Paudi Garhwal
(c) Kodagu. Korba
(d) Birhor Ranchi plateau
Explanation: B
Mudia tribe resides in Bastar district of Chhattisgarh.
32. Rajghat river project jointly established by which two states ?
(a) Uttar pradesh and Rajasthan
(b) Bihar and jharakhand
(c) Madhya pradesh and Maharashtra
(d) Uttar pradesh and Madhya Pradesh
Explanation: D
The Rajghat Dam is a joint project of the state of Uttar Pradesh and Madhya Pradesh
located on the Betwa River.
33.Which of the following pairs is not correctly mached?
(a) Kudremukh mines- iron ore
(b) Talcher - Coal
(c) Singareni - Copper
(d) Korba-Aluminum.
Explanation: C
RO Full Length Paper#1
Target PCS Lucknow Page 10
Singrauli is famous for coal production. The remaining statement is true.
34.Which of the following is a source of atmospheric energy?
(a) Infrared radiation is emitted downward from clouds.
(b) Visible radiation is scattered upward in space.
(c) Latent heat is released
(d) Albedo of earth and atmosphere.
Explanation: C
The latent heat present in the water vapor is an indicator of atmospheric energy. It is from
these that cyclones get energy.
35.Which of the following processes is important in the creation of electricity in the cloud
being formed?
(a) Absorption of solar cosmic rays
(b) Small lapse rate
(c) Intense vertical air stream
(d) Heavy rainfall
Explanation: C
Intense vertical or rising air when condensed above the condensation point creates
electricity in a cloud that causes heavy thunderstorms to rain.
36.In which of the following cities was the first City Improvement Trust established?
(a) Bombay
(b) Calcutta
(c) Hyderabad
(d) Lucknow
Explanation: A
The first City Improvement Trust of India was established in Mumbai city. This trust was
established keeping in mind the challenges of rapid growth of the Mumbai metropolis and
unplanned urbanization. The purpose of this trust was to revisit the mass transit system,
planning urban population and strengthening urban institutions. The City Improvement Trust
has also been established in other metros of the country under the Mumbai model.
37.Almost all of India's movable oil deposits are located in which of the following rocks?
(a) Ancient Granite Deposition Area
(b) Ancient Sedimentary Deposition Area
(c) Metamorphic Deposition Area
(d) Tertiary Depressed Area
Explanations: D
Almost all of India's movable oil deposits are located in sedimentary rocks dating back to the
Tertiary era. Oil deposit refers to mineral oil or petroleum or a mixture of hydrocarbon
compounds. The deposit storage of oil in India is in both offshore and onshore areas. The
major area of oil deposit in India is as follows. Brahmaputra Valley Gujarat Coast Western
Offshore Area, North Eastern Offshore Area.
38.Uttar Pradesh has a number of central universities-
(a) 5
(b) 4
RO Full Length Paper#1
Target PCS Lucknow Page 11
(c) 3
(d) 2
Explanation: B
There are a total of four central universities in Uttar Pradesh. University of Allahabad,
Banaras Hindu University, Dr. Bhimrao Ambedkar University, Lucknow and Aligarh Muslim
University.
39.Match List 1 with List 2 and choose the correct answer from the code given below the
lists -
List 1. List 2
Industry. Centre
(a) Wooden toy - Meerut
(b) Sporting goods. - Bareilly
(c) Brass sculptures. - Varanasi
(d) Matchmaker industry - Mathura
Code:. A. B. C. D
(a) 1. 4. 3. 2
(b) 3. 1. 4. 2
(c) 3. 2. 1. 4
(d) 2. 1. 4. 3
Explanation: B
Self explanatory
40.Karma folk dance is -
(a) Mahoba
(b) Poorvanchal
(c) Sonbhadra
(d) Braj region
Explanation: C
Karma folk dance is performed by Kharwar tribal people. Its area is Sonbhadra / Mirzapur.
41.Which is the famous pilgrimage place of both Jain and Buddhist religions in Uttar
Pradesh?
(a) Saranath
(b) Kaushambi
(c) Kushinagar
(d) Shravasti
Explanation: B
Kaushambi is the famous pilgrimage center for both Jain and Buddhist religions in Uttar
Pradesh, while Sarnath and Kushinagar are only related to Buddhism.
42.Renowned Thumri singer Girija Devi is related to -
(a) From Jaipur Gharana
(b) From Lucknow Gharana
(c) From Banaras Gharana.
(d) None of the above
Explanation: C
RO Full Length Paper#1
Target PCS Lucknow Page 12
Renowned Thumri singer Girija Devi is associated with the Banaras Gharana of Uttar
Pradesh, a famous music house.
43.Nawabganj Birds Vihar is located-
(a) Gonda
(b) Unnao
(c) Gaziabad
(d) Raibarelli
Explanation: B
Nawabganj Birds santuachury is located in Unnao. It was established in 1984 while Parvati
Aranga Bird Sanctuary in Gonda ; Okhla Birds sanctuary in Ghaziabad ; Samaspur Birds
santuachry in Rae Bareli .
44. First Biotech park is located in Uttar Pradesh-
(a) Lucknow
(b) Allahabad
(c) Varanasi
(d) Ghaziabad
Explanation: A
Self explanatory
45. Internal Security Academy is located -
(a) Mount abu
(b) Nasik
(c) Pune
(d) Hyderabad
Explanation: A
The Academy of Internal Security is located in Mount Abu, Rajasthan. It was established in
1975 to train officers of the Central Reserve Police Force.
46.Folk dance Rahula is associated with which one of the following regions of UP?
(a) From the eastern region
(b) From the western region,
(c) From the central region
(d) From the Bundelkhand region.
Explanation: D
The folk dance Rawala belongs to the Bundelkhand region of Uttar Pradesh. In this style of
laughing laughs, enlightening shlokas sayings, teachings are staged.
47.Rajiv Gandhi Wildlife Conservation Award is given to
(a) To Wildlife Conservatives
(b) To Forest and Wildlife Officers
(c) To Research Institutes.
(d) All of above
Explanation: D
The Rajiv Gandhi Wildlife Conservation Award is presented by the Ministry of Environment
and Forests to officers and field workers who have made significant contributions in the field
of wildlife conservation research . The award is being given since 1998.
RO Full Length Paper#1
Target PCS Lucknow Page 13
48.The main feature of the Act of 1861 was-
(a) The Governor General was empowered to issue ordinances when required.
(b) The provinces were empowered to enact laws regarding local subjects.
(c) Responsive governance was established.
(d) All the above statements are true
Explanation: A
The main feature of the Act of 1861 was. 1.Executive Council of a Governor General was
expanded. 2.Departmental system started. 3.The Governor General was given the power to
issue ordinances for the first time. 4. The Governor General was empowered to establish the
Legislative Council in Bengal, the North Western Frontier Province and Punjab.
49.The President's emergency powers have been adopted under the Indian Constitution
(a) From the Constitution of the Soviet Union
(b) Government of India Act 1919,
(c) From the Weimar Constitution of Germany,
(d) From the Constitution of the United States of America.
Explanation: C
The President of India has the power to suspend fundamental rights during the Emergency
in the Indian Constitution. She is very similar to which article of Germany's voimer
constitution. Emergency provisions are also derived from the Weimar Constitution of
Germany.
50.Which one of the following objectives is not included in the Preamble of the Constitution
of India?
(a) Freedom of thought
(b) Economic freedom
(c) Freedom of expression
(d) Freedom of belief.
Explanation: B
The Preamble of the Constitution of India provides for ensuring socio-economic and political
justice in addition to providing freedom of thought expression, faith, religion and worship to
the citizens.
51.Article 1 of the Indian Constitution declares India as a
(a) Union of states.
(b) Declares a federal state.
(c) Declares a unitary state.
(d) Declares a republican state.
Explanation: A
Article 1 of the Constitution of India declares that India, which is India, will be a union of
states, which makes it clear that the Union State is not in any way a result of the mutual
agreement of the states because any state should be separated from the Union. Does not
have authority.
52.Which one of the following is not correctly matched?
State. Article
RO Full Length Paper#1
Target PCS Lucknow Page 14
(a) Nagaland. Art.371 A
(b) Assam. Art. 371 B
(c) Meghalaya. Art. 371 C
(d) Andhra pradesh. Art.371 D
Explanations: C
Art 371 C is related to Manipur.
53. The concept of linguistic state was endorsed
(a) Nehru report
(b) Dhar commission
(c) Cabinet mission
(d) Simon commission
Explanation: B
The commission was constituted by the President of the Constituent Assembly, Dr. Rajendra
Prasad. The chairman of this commission was SK Dhar, retired judge of Allahabad High
Court. There were 4 members in this commission. The task of this Commission, especially
in South India, was to investigate the demand that states should be reorganized on the basis
of language or not. This Commission in its report opposed the reorganization of states on the
basis of language and on the basis of administrative convenience. But supported the
reorganization of states.
54.Which of the following articles in Indian Constitution is about citizenship in India?
(a) Art 333 - Art 337
(b) Art 17-Art 20
(c) Art 5 - Art 11
(d) Art 1 - Art 4
Explanation: C
There is a provision related to citizenship under Articles 5 to 11 of the Constitution. Articles
1 to 4 have provisions relating to the Union and its territory and Article 17 ends
untouchability. Article 18 ends the titles. Article 19 makes provisions for protection of
certain rights of speech and expression and Article 20 in relation to conviction for offenses.
55.Which of the statements is correct?
(a)The Constitution of India is presidency.
(b)India is a nominal monarchy.
(c) India is an oligarchy.
(d)India is a communal democracy.
Explanation:D
The expression of the Preamble of the Constitution makes it clear that the Constitution of
India has recognized the ultimate source of political power to the people. According to the
constitution, the cabinet will be answerable before the house elected by the public and the
second part of the executive ie the President is also responsible before the Parliament in the
sense that in certain circumstances the Parliament can remove it by impeachment.
56.One point at which equality is found in Indian and American federalism is:
(a) the residual powers to be near the center.
(b) Residual powers possess states.
(c) In some cases there is a provision for concurrent area.
RO Full Length Paper#1
Target PCS Lucknow Page 15
(d) In disputes between states, the Supreme Court resolves the dispute.
Explanation: D
The American federal system and the Indian federal system means that the right has been
divided between the union and its units in both countries and the Supreme Court acts as a
decider if a dispute arises between the union and its units.
57.Which of the following is stated in the Constitution of India?
(a) President will not be a member of any House of Parliament.
(b) The Parliament will consist of the President and two Houses.
Codes:
(a) Only 1
(b) Only 2
(c) 1 and 2 both
(d) None of the above
Explanation: C
Article 79 of the Constitution of India provides that there shall be an MP for the Union, whose
President shall consist of the Rajya Sabha and the Lok Sabha. The President shall not be a
member of either House on Parliament.
58. If any money bill is approved by the Lok Sabha, then Rajya Sabha can stop it at most-
(a) 6 month
(b) 4 month
(c) 1 month
(d) 14 days
Explanation: D
Article 110 of the constitution states that a bill will be a money bill only if the speaker of the
Lok Sabha gives a certificate of its being. The money bill is passed in the Lok Sabha and
sent to the Rajya Sabha. If the Rajya Sabha does not take any action within 14 days, it is
presumed that it has accepted the bill and will be deemed to have been passed by both the
Houses. But if an amendment is passed by the Rajya Sabha, the Money Bill can accept or
reject the recommendations of the Lok Sabha.
59.How does the Indian Parliament control the administration?
(a) Through Parliamentary committees.
(b) Through the consultancy committees of various ministries.
(c) Send periodic reports to the administrators.
(d) Forcing the executive to issue writ.
Explanation:A
The Parliament of India controls the administration through committees. For example, it
controls public finance through 3 committees. Public Accounts Committee, Estimates
Committee, Committee on Public Undertakings.
60.Under which article of Indian constitution can the Governor reserve a bill for the
consideration of the President?
(a) Art 169
(b) Art 200
(c) Art 201
(d) Art 257
RO Full Length Paper#1
Target PCS Lucknow Page 16
Explanation: B
Under Article 200, the Governor can reserve any bill passed by the State Legislature for
consideration of the President.
61.How long can a member of the Council of Ministers stay in his office without becoming a
member of the state assembly?
(a) 1 years
(b) 3 years
(c) 6 month
(d) 3 month
Explanation: C
According to Article 164, no Minister who is not a member of the State Legislature for a
continuous period of 6 months shall cease to be a Minister at the end of this period.
62.In which of the following years was the Federal Court of India established?
(a) 1935
(b) 1934
(c) 1936
(d) 1937
Explanation: D
The Federal Court in India was established in 1933 under the Government of India Act,
1935.
63.Panchayats were granted constitutional status-
(a) Art 226
(b) Art 239
(c) Art 243
(d) Art 219
Explanation: C
Constitutional status has been accorded to Panchayats under Article 243. A provision has
been made in relation to Panchayats under Article 243 of Part 9 of the Constitution.
64.The state government does not have control over local units in any of the following areas
(a) Citizen's complaints.
(b) Economic matters.
(c) Law formation.
(d) Personnel matters
Explanation:A
The 11th schedule has been added to the Constitution by 73 constitutional amendments.
The provisions related to Panchayati Raj institutions have been included in this. In this, state
governments have been given authority over local bodies over certain areas. Personnel law
is important in the economic field, etc., but excludes citizen complaints.
65.The State Election Commission of Uttar Pradesh is -
(a) a statutory body.
(b) An executive body.
(c) A constitutional body.
(d) A unit of the Election Commission of India.
RO Full Length Paper#1
Target PCS Lucknow Page 17
Explanation: C
Article 324 provides for an Election Commission for India. The Election Commission is a
constitutional and all-India institution as it is common to both the central and state
governments.
66.Which instrument is used to measure height?
(a) Ammeter
(b) Anemometer
(c) Hydrometer
(d) Altimeter.
Explanation: D
Altimeter is a height measuring instrument mainly used in planes. Hydrometer is a
comparative density measuring device of water. The ammeter is an electric current
measuring device. Anemometer is an air velocity measuring device.
67.What are the atoms that have the same number of protons but different numbers of
neutrons?
(a) Isobars
(b) Isomers
(c) Isotherms
(d) Isotopes
Explanation: D
Atoms of the same element with the same atomic number but different mass numbers are
called isotopes.The reason why an element has the same atomic number of different
isotopes is that they have the same number of protons in their nucleus, but the number of
neutrons in them varies, due to which their mass numbers vary.
68.Which of the following is the basic element of carbon?
(a) Sand
(b) Diamond
(c) Marble
(d) Suger
Explanation: B
Diamond is chemically the purest form of carbon. So it is a basic element. The sand is
mainly silicon and oxygen. Marble, calcium carbon and oxygen. And sugar is mainly a
mixture of carbon, hydrogen and oxygen.
69.Aluminum pages are often anodized. It means depositing on it -
(a) Chromium oxide layer
(b) Aluminum oxide layer
(c) Nickel oxide layer
(d) Zinc oxide layer
Explanation: B
Anodizing is the electrochemical process by which a metal surface is made durable and
corrosion resistant. In this process, aluminum oxide layer is deposited on aluminum.
70.Which of the following does not contain carbon? (a)Diamond
(b) graphite
RO Full Length Paper#1
Target PCS Lucknow Page 18
(c)Coal
(d) Sand
Explanation: D
diamond is the crystalline form of carbon. Its relative density is 3.5. Graphite is a crystalline
form of carbon that is a smooth shiny black color while coal is a solid organic material that is
used as a fuel.
71.Which of the following gases is used to purify drinking water?
(a) Carbondioxide
(b) Fluorine
(c) Chlorine
(d) Helium
Explanation: C
The Chlorine is used as a disinfectant for making bleaching powder and as a germicide in
drinking water.
72.The chemical name of blue thoth is-
(a) Sodium bicarbonate
(b) Sodium hydroxide
(c) Magnesium sulfate
(d) Copper sulfate.
Explanation: D
Blue Thoth is Copper Sulphate ;Epsom Salt is Magnesium Sulphate ;Baking Soda is Sodium
Bicarbonate ;Caustic Soda is Sodium hydroxide.
73.Which of the following promotes the ripening of fruits?
(a) Carbon dioxide
(b) Sulfur dioxide
(c) Nitrogen
(d) Ethylene
Explanation: D
Ethylene gas is used in ripening of fruits.
74. What gas is produced in plastic?
(a) Polynitrogen
(b) Polyhydrones
(c) Polychlorin
(d) Polyethylene.
Explanation: D
Polyethylene gas is produced from plastic
75.Which of the following is found in lactic acid,
(a) Lemon
(b) Butter
(c)Milk
(d) Vinegar
Explanation: C
RO Full Length Paper#1
Target PCS Lucknow Page 19
Lactic acid in milk is found in acetic acid in vinegar and citric acid in lemon. Butyric acid is
found in butter.
76.Which one of the following is not a Ape? (a) Gibbon
(b)Gorilla,
(c)Langur
(d)Orang Utan
Explanation: C
The langur is not included in the Ape but in monkey.
77.Which of the following vitamins is called Riboflavin?
(a) B1
(b) B2
(c) B6
(d) C
Explanation: B
The chemical name of B1 is thiamine. Due to its deficiency, there is a disease called Berry
Berry. Riboflavin to Vitamin B2. Pyridoxine to vitamin B6. Vitamin C is called ascorbic acid.
78.The National Academy of Agricultural Research Management is located.
(a) Bengaluru
(b) Hyderabad
(c) Kolakata
(d) New delhi
Explanation: B
79.Which of the following fruits is found in abundant quantity?
(a) Jamun
(b) Karaunda
(c) Lokat
(d) Guava.
Explanation: B
80.Where is Nauchandi fair held in Uttar Pradesh?
(a) Varanasi
(b) Pryagraj
(c) Meerut
(d) Saharanpur
Explanation: C
81.According to Indian Council of Agricultural Research, India has number of agricultural
ecological zones?
(a) 15
(b) 20
(c) 17
(d) 18
Explanation: B
RO Full Length Paper#1
Target PCS Lucknow Page 20
82.Total number of census towns in India as per 2011 census-
(a) 3894
(b) 4041
(c) 5161
(d) 7935
Explanation: D
As per the 2011 population, the total number of census towns in India was 7935 while in
2001 they were 5161.
83.In which city is the Agro Processing Zone located in Uttar Pradesh?
(a) Unnao
(b) Saharanpur
(c) Kanpur
(d) Lucknow
Explanation: B
The Leather Technology Park is in Unnao. Agro Processing Zone is located in Saharanpur,
Bio Technology Park in Lucknow, Apparel Park in Kanpur.
84.In Which states locusts enter India?
(a) Gujrat
(b) Maharastra
(c) West Bengal
(d) Rajasthan
Explanation: D
Locuts enters Rajasthan state from Pakistan in India.
85.The Human Development Report is published globally every year:
(a) UNDP
(b) WTO
(c) IMF
(d) WORLD BANK
Explanation: A
UNDP (United Nations Development Programme)
86.Which of the following is not included in the 8 goals of the Climate Action Plan of the
Government of India?
(a) Solar Power
(b) Consumer
(c) Nuclear
(d) Housing Area.
Explanation: C
The 8 goals of the Climate Action Plan of the Government of India include the National Solar
Mission, National Enhanced Energy Saving Mission, National Sustainable Habitat Mission,
National Water Mission, National Himalaya Ecological Preservation Mission, National Green
India Mission, National Sustainable Agriculture Mission, National Climate Change Strategic
Knowledge Mission.
Hence it is clear that it does not include nuclear power.
RO Full Length Paper#1
Target PCS Lucknow Page 21
87.In which year was the Water Pollution Prevention and Control Act implemented?
(a) 1986
(b) 1981
(c) 1974
(d) 1972
Explanation: C
The Central Pollution Control Board of India was formed in September 1974 under the Water
Pollution Prevention and Control Act 1974 as a statutory organization. It provides technical
services to the Ministry of Environment and Forests under the provisions of the Environment
Protection Act 1986.
88.Which of the following is not a water borne disease?
(a) Typhoid
(b) Hepatitis
(c) Cholera
(d) Dengue
Explanation: D
Typhoid, hepatitis is a waterborne disease. Dengue is caused by a type of virus infection
that is spread by a fly called Aedes aegypti.
89.Sort the following National Parks of India from north to south direction:
(a) Indravati National Park,
(b) Nagarhole National Park
(c) Corbett National Park
(d) Madhav National Park.
Codes:
(a) 2,3,4,1
(b) 2,1,4,3
(c) 1,3,4,2
(d) 3,4,1,2
Explanation: D
Corbett National Park, Uttarakhand; Madhav National Park, Madhya Pradesh; Indravati
National Park, Chhattisgarh; Nagarhole National Park, Karnataka.
90.Which one of the following is a source of methane emission in the atmosphere?
(a) Automatic vehicle vacuum sputter
(b) Industrial chimney
(c) Mining
(d) Wetlands.
Explanation: D
Methane gas emissions predominantly from leaks in wetlands such as rice fields, livestock,
land, and natural gas systems or account for 11% of greenhouse emissions.
91.In geographical areas, Kanha National Park is related to which one?
(a) Tropical low wet forest
(b) Tropical wet forest
(c) Tropical dry forest
(d) Subtropical forest
RO Full Length Paper#1
Target PCS Lucknow Page 22
Explanation: A
The biographical areas of Kanha National Park are related to tropical low humid forest
92.The range of the number of animals in a certain area that the environment can support is
called the
(a)Population.
(b) Carrying capacity
(c)Number or the pyramid of bio mass
(d)None of the above.
Explanation: B
93.Nitrogen oxide is the major source of pollution in urban areas
(a) Energy power plants
(b) Road transport
(c) Commerce sector
(d) Industries.
Explanation: B
Nitrogen oxide originates from the burning of mineral oil and coal. Due to excessive
concentration of nitric oxide in the human body and many diseases it suffers from diseases.
Such as gingivitis, bleeding, pneumonia, lung cancer
94.When is Biodiversity Day celebrated?
(a) 28 Feburary
(b) 29 December
(c) 27 June
(d) 22 May
Explanation: B
Biodiversity Day is celebrated by United Nations on 22 May, while many countries of the
world celebrate Biodiversity Day on 29 December. It is noteworthy that the Convention on
Biodiversity was effective from 29 December 1993.
95. YASH Program, recently in the news, is related to:
(a) Gold monetisation
(b) Health and risk communication
(c) Employment for rural workers amid Covid-19 outbreak
(d) Welfare measures for defence personnel
Explanation: B
The National Council for Science & Technology Communication (NCSTC), Department
of Science & Technology (DST) has launched a programme on health and risk
communication ‘Year of Awareness on Science & Health (YASH)’ with focus on Covid-
19.The programme is a comprehensive and effective science and health communication
effort for promoting grass-root level appreciation and response on health.
96.When did the World Trade Organisation come into effect?
RO Full Length Paper#1
Target PCS Lucknow Page 23
(A) March 6, 1996
(B) April 8, 1994
(C) February 5, 1994
(D) January 1, 1995
Explanation: D
The World Trade Organization (WTO) is an intergovernmental organization that is
concerned with the regulation of international trade between nations. The WTO officially
commenced on 1 January 1995 under the Marrakesh Agreement, signed by 123 nations on
15 April 1994, replacing the General Agreement on Tariffs and Trade (GATT), which
commenced in 1948. It is the largest international economic organization in the world.
97.Mount Harriet National Park, recently in the news, is located in:
(a) Karnataka
(b) Dada and Nagar Haveli
(c) Goa
(d) Andaman and Nicobar Islands
Explanation: D
Mount Harriet National Park is a national park located in the Andaman and Nicobar Islands
union territory of India. The park, established in 1969.
98.Which Indian state/UT has formed a dedicated commission to create employment
opportunities for the migrant labourers?
(a) Odisha
(b)Madhya Pradesh
(c)Uttar Pradesh
(d)Bihar
Explanation: B
The state government of Madhya Pradesh has formed a dedicated commission to create
employment opportunities for the migrant labourers.
Named as the Madhya Pradesh State Migrant Labourer Commission (MPSMLC), it aims to
create jobs to those who have returned from other parts of the country during the Covid 19-
induced lockdown.
99.What is the theme of the ‘International Day of Parliamentarism’ 2020?
(a)Parliaments in a time of pandemic
(b) Role of Parliaments
(c)Democracy and Parliament
(d) Parliaments Matter
Explanation: A [Parliaments in a time of pandemic]
June 30 is the day designated by the United Nations to celebrate the International Day of
Parliamentarism.
RO Full Length Paper#1
Target PCS Lucknow Page 24
The United Nations General Assembly recognized the role of parliaments in developing the
world, and designated the special day. On the same day in the year 1889, the global
organization of parliaments called the Inter-Parliamentary Union (IPU) was established.
100.The term “SPICe”, sometimes seen in news, is related to which of the following fields?
(a) Science & Technology
(b)Environment Protection
(c) Business & Economy
(d) Defence and Arms Production
Explanation: C
Ministry of Corporate Affairs is replacing the existing SPICe (Simplified Proforma for
Incorporating Company Electronically) form with a new web form called SPICe+.
101.Who is the head of the High level expert group on farm exports, recently set up by the
15th Finance Commission?
(a) N K Singh
(b) U K Sinha
(c) Tapan Ray
(d) Sanjiv Puri
Explanation: D
The 15th Finance Commission (FFC) recently set up a High level expert group on farm
exports. The group is to be headed by the Chairman and Managing director of ITC, Sanjiv
Puri.The committee will suggest performance-based incentives to the state governments for
2021-22 to 2025-26, to accelerate agricultural reforms and exports in the sector.
102.In which year Government of India started Navratna category?
(a)1992
(b) 1993
(c) 1995
(d) 1997
Explanation: D (1997)
The Public Sector Enterprises are run by the Government under the Department of Public
Enterprises of Ministry of Heavy Industries and Public Enterprises. The government grants
the status of Navratna, Miniratna and Maharatna to Central Public Sector Enterprises based
upon the profit made by these CPSEs. The Maharatna category has been the most recent
one since 2009, other two have been in function since 1997.
103.The portion of profits that a company distributes among its shareholders in the form of
cash is usually known as _____?
(a) Yield
(b) Dividend
(c) Stock Split
(d) Free Float
Explanation: B
Dividend is the portion of profits that a company distributes among its shareholders in the
form of cash. Usually it is expressed per share. In some cases it is expressed as a
percentage of the share`s face value.
RO Full Length Paper#1
Target PCS Lucknow Page 25
104.What is the number of indicators in Multi-dimensional Poverty Index (MPI) ?
(a) 8
(B)9
(c) 10
(d) 11
Explanation: C
Dimension Indicators Deprivation Cutoffs
[10][11]
Health Child
mortality
Deprived if a child under the age of 18 years has died in the family
In the 5 years preceding the survey.
Nutrition Deprived if any adult or child, for whom there is nutritional
information, is undernourished.
Education Years of
schooling
Deprived if no household member has completed six years of
schooling.
School
attendance
No household member aged 'school entrance age + six' years or
older has completed six years of schooling.
Standard of
living
Cooking
fuel
Deprived if the household cooks with dung, wood or charcoal.
Sanitation Deprived if the household's sanitation facility is not improved
(according to MDG guidelines), or it is improved but shared with
other households.
Drinking
water
Deprived if the household does not have access to improved
drinking water (according to MDG guidelines) or improved drinking
water is more than a 30-minute walk from home round trip.
Electricity Deprived if the household has no electricity.
Housing Deprived if at least one of the three housing materials for roof,
walls and floor are inadequate: the floor is of natural materials
and/or the roof and/or walls are of natural or rudimentary
RO Full Length Paper#1
Target PCS Lucknow Page 26
materials.
Assets Deprived if the household does not own more than one of these
assets: radio, TV, telephone, computer, animal cart, bicycle,
motorbike or refrigerator and does not own a car or truck
105.In India, Banks can accept interest free deposits in ___
(a)Current Account
(b)Saving Account
(c)Demand Deposits
(d)Term Deposits
Explanation: A
Banks cannot accept interest free deposits other than in current account.
106.Which of the following is / are auctioned in Open Market Operations?
(a) Shares
(b) Debentures
(c) Securities
(d) Bullion
Explanation: C
Open Market Operations refer to the purchase and sale of the Government securities (G-
Secs) by RBI from / to market.
107.Which Investment bank collapse started the 2008 global financial crisis?
(a) Morgan Stanley
(b)Goldman Sachs
(c)Lehman Brothers
(d) Merill Lynch
Explanation: C
The collapse of Lehman Brothers in 2008 has started the crisis in 2008. The bank filed
bankruptcy after it failed to repay its payments. It was an investment banking firm.
108.Sahyog-Kaijin’ is an annual joint exercise between the Coast Guards of India and which
country?
(a) Bangladesh
(b) China
(c) Japan
(d) Sri Lanka
Explanation:C
Sahyog-Kaijin’ is an annual joint exercise between the Coast Guards of India and Japan.
The 19th edition of the exercise is scheduled to begin in Chennai, India.
RO Full Length Paper#1
Target PCS Lucknow Page 27
109.What is the theme of the DefExpo 2020, the 11th edition of the biennial defence
exhibition?
(a)India: The Emerging Defence Manufacturing Hub
(b)Artificial Intelligence in Defence
(c) Make in India- A pride in Defence
(d) Digital battlefield of the future
Explanation: A
The 11th edition of the biennial mega defence exhibition, DefExpo 2020 is to be held in the
capital city of Uttar Pradesh- Lucknow from February 5 to 9, 2020. The theme of this edition
of the event is ‘India: The Emerging Defence Manufacturing Hub’. The sub-theme is
announced as ‘Digital Transformation of Defence’.
110.According to the recent Economic Survey 2019-20, which state has the cheapest
Vegetarian Thali (food plate)?
(a) Kerala
(b) Jharkhand
(c) Bihar
(d) Tripura
Explanation: B
The term called ‘Thalinomics’ was used, which means the affordability of buying a Thali
(Food Plate).
It states that, Jharkhand has the cheapest Vegetarian Thali (meals plate), less than Rs.50.
111.Which Indian businessperson has entered the top-10 of Forbes India Rich List 2020 for
the first time?
(a) Gautam Adani
(b) Shiv Nadar
(c)Cyrus Poonawalla
(d) Uday Kotak
Explanation: C
As per the Forbes India Rich List 2020 that was released recently, the chairman of Reliance
Industries Limited Mukesh Ambani remains the richest Indian for the 13th consecutive
year.His net worth stands at USD 88.7 billion.The new entrant in the top 10 is Cyrus
Poonawalla, the Chairman of Serum Institute of India (SII).
112.Who has been recently awarded with the Subash Chandra Bose Aapda Prabandhan
Puraskar, 2020?
(a) Vijay Kumar
(b) Sylendra Babu
(c) Kumar Munnan Singh
(d) Naveen Kumar
Explanation: C
Kumar Munnan Singh has been selected for the Subhash Chandra Bose Aapda Prabandhan
Puraskar 2020 under the Individual category. Under the Institution category, Disaster
Mitigation & Management Centre, Uttarakhand has been selected for the award. Kumar
RO Full Length Paper#1
Target PCS Lucknow Page 28
Munnan Singh was the founding member of the National Disaster Management Authority in
2005. He is also lauded for establishing the National Disaster Response Force (NDRF).
113.Which Indian sportsperson is set to take charge as the President of the Paralympic
Committee of India?
(a) Devendra Jhajharia
(b) Deepa Malik
(c) Mariyappan Thangavelu
(d) Varun Singh Bhati
Explanation: B
india’s first and only woman Paralympics silver medallist, Deepa Malik is set to take charge
as the President of the Paralympic Committee of India.
114.‘World Consumer Rights Day’, 2020 is -
(a) 12 March
(b) 13 March
(c) 14 March
(d) 15 March
Explanation: D
World Consumer Rights Day is celebrated every year on March 15 across the world to
spread awareness about the rights and needs of a consumer. Theme of this year is
“Sustainable Consumer”.
115.When did this year’s Earth Overshoot Day fall?
(a) August 20
(b) August 22
(c) June 19
(d) June 22
Explanation: B
This year, the Earth Overshoot Day fell on 22nd August, 2020. Earth Overshoot Day is the
date when the demand for ecological resources and services by human beings exceeds
what Earth can regenerate in a given year.This is computed by the think tank named Global
Footprint Network.
116.Axone, a popular condiment of Nagaland is made from which of the following?
(a) Rice
(b) Wheat
(c) Corn
(d) Soya bean
Explanation: D
117.Global Economic Prospects report is released by?
(a) World Bank
(b) World Trade Organisation
(c) International Monetary Fund
(d) World Economic Forum
Explanation:A
RO Full Length Paper#1
Target PCS Lucknow Page 29
118.Which numbers will come next in the following series?
21 11 6 3.5 ?
(a) 2.5 (b)1.5 (c)2 (d)2.25
Explain- D
21+1 and divided by 2=11
11+1 and divided by 2=6
So 3.5+1 and divided by 2 = 2.25
119. Choose the correct option..
54:9::?:7
(a) 63
(b) 42
(c) 21
(d) 28
Explain (B)
9×6=54 so 7×6=42
120. Which one of the following number is different from other?
(a) 96
(b) 87
(c) 69
(d) 68
Explain (D)
Because sum of the number is 15
121. Which one of the following is different from other three?
RO Full Length Paper#1
Target PCS Lucknow Page 30
(a) Shyam: honest
(b) Ram: loyal
(c) Sita:lazy
(d) Sunder:dark face
Explain ( D)
dark face is god gifted
122. In a certain code if
RAM = 321
NAVI = 6245
HARIOM = 92381
MOHAN = ??
(a) 54321
(b) 19876
(c) 18926
(d) 89678
Explain (C )
number used as word code
123. As Sita is related to Ram then in which of them related to shyam??
(a) Dwarika
(b) Radha
(c) Sudama
(d) Gwal
Explain (B)
used as god
RO Full Length Paper#1
Target PCS Lucknow Page 31
124. Which one of the following is different from other three?
(a) HI = 17
(b) MN = 13.5
(c) PQ = 16.5
(d) ZS = 22.5
Explain (A)
sum of the place value of the word and divided by 2
125. Which one of them is not created from 'INFORMATION'?
(a) FROM
(b) FOR
(c) MENTION
(d) MOTION
Explain (C)
E is not in INFORMATION
126. Fill the blanks..
COMP_ _ _ _
(a) OTER
(b) UTER
(c) UTAR
(d) UTRE
Explain (B)
COMPUTER
RO Full Length Paper#1
Target PCS Lucknow Page 32
127. Which one of the following is different from other three?
(a) 9×11=99
(b) 8×9=72
(c) 14×12=84
(d) 16×14=64
Explaination :(D)
LCM of the number
128.CogX, a global leadership summit, is related to:
(a) Artificial Intelligence & Emerging Technology
(b) Preservation of Corals
(c) Space Junk
(d) Freedom of Press
Explanation: A
CogX is a prestigious Global Leadership Summit and Festival of Artificial intelligence &
Emerging Technology held annually in London with over 15,000 participants in attendance
from the highest levels of business, government, industry, and research.
129.The ‘Tangam language’ which is marked ‘critically endangered’ by UNESCO is spoken
in which state?
(a) Kerala
(b) Chattisgarh
(c) Arunachal Pradesh
(d) Mizoram
Explanation: C
tangam is a little-known community within the larger Adi tribe of Arunanchal pradesh which
resides in the hamlet of Kugging in Upper Siang district’s Paindem circle.For long, the
only account of the Tangams could be found in a book, “Tangams” (1975) where the
community’s population was pegged at 2,000 spread across 25 villages.
130. In which year The Central Drug Reaserch Institute was established
(a) 1949
(b) 1950
(c) 1951
(d) 1952
Explanation: C
The Central Drug Research Institute was one of the first laboratories to be established in
India right after its independence. It is among the thirty nine laboratories that are functioning
under the aegis of the council of scientific and Industrial Research Council of Scientific and
Industrial Research of India. The research institute was formally inaugurated on 17 February
1951 by Prime Minister, Jawahar Lal Nehru.
RO Full Length Paper#1
Target PCS Lucknow Page 33
131.. Recently in news, the ‘Peace Forest Initiative’ has been launched by:
(a) South Korea
(b) India
(c) Brazil
(d) Bhutan
Explanation: A
Peace Forest Initiative: It is an initiative of South Korea to use ecological restoration as a
peace-building process. It aims at addressing the issue of land degradation in conflict-torn
border areas and would go a long way in alleviating tensions and building trust between
communities living there and between enemy countries in particular.
132.Which of the following water bodies borders Saudi Arabia:
1. Red Sea
2. Gulf of Aden
3. Persian Gulf
4. Mediterranean Sea
Choose the correct option:
(a) 1 and 3 only (b) 2 and 4 only (c) 2, 3 and 4 only (d) 1, 3 and 4 only
Explanation: A
133.The first census was conducted in India -
(a) 1861
(b) 1972
(c) 1883
(d) 1881
Explanation: D
A systematic and modern population census, in its present form was conducted non
synchronously between 1865 and 1872 in different parts of the country. This effort
culminating in 1872 has been popularly labeled as the first population census of India
However, the first synchronous census in India was held in 1881. Since then, censuses have
been undertaken uninterruptedly once every ten year.
134.Danakil Depression is located in which of the following country?
(a) Australia
(b) Yemen
(c) Ethiopia
(d) Russia
Explanation: C
The Danakil Depression is the northern part of the Afar Triangle or Afar Depression in
Ethiopia, a geological depression that has resulted from the divergence of three tectonic
plates in the Horn of Africa.
135.What is the correct descending order of Rice producing states in india?
(a) Punjab, West Bengal, Uttar Pradesh And Andhra Pradesh
(b) Punjab, West Bengal, Uttar Pradesh And Andhra Pradesh
(c) Punjab, West Bengal, Uttar Pradesh And Andhra Pradesh
(d) West Bengal, Uttar Pradesh, Punjab, And Andhra Pradesh
RO Full Length Paper#1
Target PCS Lucknow Page 34
Explanation: D
Top Rice Producing States: West Bengal > Punjab > Uttar Pradesh > Andhra Pradesh >
Bihar. It is the staple food crop of majority of Indian people. India is the second largest
producer of rice in the world after China. In states like Assam, West Bengal and Odisha,
three crops of paddy are grown in a year.
136.Who approves the Fair and Remunerative Price (FRP) of sugarcane?
(a) Cabinet Committee on Economic Affairs
(b) Commission for Agricultural Costs and Prices
(c) Directorate of Marketing and Inspection, Ministry of Agriculture
(d) Agricultural Produce Market Committee
Explanation: A
The Federal/Central Government announces Fair and Remunerative Prices which are
determined on the recommendation of the Commission for Agricultural Costs and
Prices (CACP) and are announced by the Cabinet Committee on Economic Affairs,
which is chaired by Prime Minister.The State Advised Prices (SAP) are announced by key
sugarcane producing states which are generally higher than FRP.
137.The persons who became the Chief Minister for the longest period in India ?
(a) E.K. Nayanar
(b) Vasantha Sate
(c) Jyothi Basu
(d) K. Karunakaran
Explanation: C
Jyoti Basu was an Indian politician and statesman who served as the Chief Minister of West
Bengal state from 1977 to 2000
138. In which place the first Medical college was established in India ?
(a) Kolkota
(b) Agra
(c) Bangalore
(d) Delhi
Explanation: A
Calcutta Medical College, officially Medical College and Hospital, Kolkata, is an Indian
medical school and hospital. The school was established in 1835 by Lord William Bentinck
as Medical College, Bengal during British Raj.
RO Full Length Paper#1
Target PCS Lucknow Page 35
139.Who introduced English education in India ?
(a) Cornwalis
(b) Rippon
(c) Delhousie
(d) William Bentinck
Explanation: D
140.Van Vihar National Park, recently seen in news, is located in
(a) Andhra Pradesh
(b) Meghalaya
(c) Odisha
(d) Madhya Pradesh
Explanation: D
Van Vihar National Park is a national park in Bhopal, the capital city of Madhya Pradesh in
central India. Declared a national park in 1979

More Related Content

Similar to UPPSC Review Officer (RO/ARO) Test Series full-length paper#1 English Solution - Target PCS Lucknow

(Www.entrance exam.net)-acio ib exam paper-2
(Www.entrance exam.net)-acio ib exam paper-2(Www.entrance exam.net)-acio ib exam paper-2
(Www.entrance exam.net)-acio ib exam paper-2
prerana Prajapat
 
ASSESSMEN1 8th class.docx
ASSESSMEN1 8th class.docxASSESSMEN1 8th class.docx
ASSESSMEN1 8th class.docx
JaveriaFathima7
 
Sslc social-5-model-question-papers-english-medium
Sslc social-5-model-question-papers-english-mediumSslc social-5-model-question-papers-english-medium
Sslc social-5-model-question-papers-english-mediummohanavaradhan777
 
Sslc social-science-em-model-question-paper1
Sslc social-science-em-model-question-paper1Sslc social-science-em-model-question-paper1
Sslc social-science-em-model-question-paper1mohanavaradhan777
 
Sslc social-science-em-model-question-paper
Sslc social-science-em-model-question-paperSslc social-science-em-model-question-paper
Sslc social-science-em-model-question-papermohanavaradhan777
 
Class 10-social-em-model-question-2011-12-1
Class 10-social-em-model-question-2011-12-1Class 10-social-em-model-question-2011-12-1
Class 10-social-em-model-question-2011-12-1Ranganathan Nagendran
 
CLASS VIII SLATE SOCIAL WITH KEY
CLASS VIII SLATE SOCIAL WITH KEY CLASS VIII SLATE SOCIAL WITH KEY
CLASS VIII SLATE SOCIAL WITH KEY
Rc Os
 
Unit 3 Examination106GED 130 Introduction to Civiliza.docx
Unit 3 Examination106GED 130  Introduction to Civiliza.docxUnit 3 Examination106GED 130  Introduction to Civiliza.docx
Unit 3 Examination106GED 130 Introduction to Civiliza.docx
dickonsondorris
 
Target PSC Lucknow Sectional Paper#1 English Solution Sample - UPPSC/UPPCS P...
Target PSC Lucknow Sectional Paper#1 English Solution Sample -  UPPSC/UPPCS P...Target PSC Lucknow Sectional Paper#1 English Solution Sample -  UPPSC/UPPCS P...
Target PSC Lucknow Sectional Paper#1 English Solution Sample - UPPSC/UPPCS P...
Target PCS Lucknow
 
GR - I TAF MASTER TEST BATCH LEVEL - 2 (REVISION - 1) BILINGUAL WITHOUT ANS (...
GR - I TAF MASTER TEST BATCH LEVEL - 2 (REVISION - 1) BILINGUAL WITHOUT ANS (...GR - I TAF MASTER TEST BATCH LEVEL - 2 (REVISION - 1) BILINGUAL WITHOUT ANS (...
GR - I TAF MASTER TEST BATCH LEVEL - 2 (REVISION - 1) BILINGUAL WITHOUT ANS (...
ramji2015
 
Practice Stimuli Response Questions for World History
Practice Stimuli Response Questions for World HistoryPractice Stimuli Response Questions for World History
Practice Stimuli Response Questions for World History
Matthew Caggia
 
Quiz ppt, how quiz helps in learning english
Quiz ppt, how quiz helps in learning englishQuiz ppt, how quiz helps in learning english
Quiz ppt, how quiz helps in learning english
justinroy26
 
INDIA@75 QUIZ_QUIZZITCH CUP 2023
INDIA@75 QUIZ_QUIZZITCH CUP 2023INDIA@75 QUIZ_QUIZZITCH CUP 2023
INDIA@75 QUIZ_QUIZZITCH CUP 2023
Anand Kumar
 
General knowledge
General knowledge General knowledge
General knowledge
Kendral Flores
 
3. Penulisan soal HOTS.pdf
3. Penulisan soal HOTS.pdf3. Penulisan soal HOTS.pdf
3. Penulisan soal HOTS.pdf
BintangPangalambohiS
 
Quiz Competition : MCQ
Quiz Competition : MCQQuiz Competition : MCQ
Quiz Competition : MCQ
Prashant Arsul
 

Similar to UPPSC Review Officer (RO/ARO) Test Series full-length paper#1 English Solution - Target PCS Lucknow (20)

(Www.entrance exam.net)-acio ib exam paper-2
(Www.entrance exam.net)-acio ib exam paper-2(Www.entrance exam.net)-acio ib exam paper-2
(Www.entrance exam.net)-acio ib exam paper-2
 
Alit amriani
Alit amrianiAlit amriani
Alit amriani
 
ASSESSMEN1 8th class.docx
ASSESSMEN1 8th class.docxASSESSMEN1 8th class.docx
ASSESSMEN1 8th class.docx
 
Sslc social-5-model-question-papers-english-medium
Sslc social-5-model-question-papers-english-mediumSslc social-5-model-question-papers-english-medium
Sslc social-5-model-question-papers-english-medium
 
Sslc social-science-em-model-question-paper1
Sslc social-science-em-model-question-paper1Sslc social-science-em-model-question-paper1
Sslc social-science-em-model-question-paper1
 
Sslc social-science-em-model-question-paper
Sslc social-science-em-model-question-paperSslc social-science-em-model-question-paper
Sslc social-science-em-model-question-paper
 
Class 10-social-em-model-question-2011-12-1
Class 10-social-em-model-question-2011-12-1Class 10-social-em-model-question-2011-12-1
Class 10-social-em-model-question-2011-12-1
 
CLASS VIII SLATE SOCIAL WITH KEY
CLASS VIII SLATE SOCIAL WITH KEY CLASS VIII SLATE SOCIAL WITH KEY
CLASS VIII SLATE SOCIAL WITH KEY
 
Unit 3 Examination106GED 130 Introduction to Civiliza.docx
Unit 3 Examination106GED 130  Introduction to Civiliza.docxUnit 3 Examination106GED 130  Introduction to Civiliza.docx
Unit 3 Examination106GED 130 Introduction to Civiliza.docx
 
General knowledge
General knowledgeGeneral knowledge
General knowledge
 
Target PSC Lucknow Sectional Paper#1 English Solution Sample - UPPSC/UPPCS P...
Target PSC Lucknow Sectional Paper#1 English Solution Sample -  UPPSC/UPPCS P...Target PSC Lucknow Sectional Paper#1 English Solution Sample -  UPPSC/UPPCS P...
Target PSC Lucknow Sectional Paper#1 English Solution Sample - UPPSC/UPPCS P...
 
Social science
Social scienceSocial science
Social science
 
GR - I TAF MASTER TEST BATCH LEVEL - 2 (REVISION - 1) BILINGUAL WITHOUT ANS (...
GR - I TAF MASTER TEST BATCH LEVEL - 2 (REVISION - 1) BILINGUAL WITHOUT ANS (...GR - I TAF MASTER TEST BATCH LEVEL - 2 (REVISION - 1) BILINGUAL WITHOUT ANS (...
GR - I TAF MASTER TEST BATCH LEVEL - 2 (REVISION - 1) BILINGUAL WITHOUT ANS (...
 
Practice Stimuli Response Questions for World History
Practice Stimuli Response Questions for World HistoryPractice Stimuli Response Questions for World History
Practice Stimuli Response Questions for World History
 
Gk scores
Gk scoresGk scores
Gk scores
 
Quiz ppt, how quiz helps in learning english
Quiz ppt, how quiz helps in learning englishQuiz ppt, how quiz helps in learning english
Quiz ppt, how quiz helps in learning english
 
INDIA@75 QUIZ_QUIZZITCH CUP 2023
INDIA@75 QUIZ_QUIZZITCH CUP 2023INDIA@75 QUIZ_QUIZZITCH CUP 2023
INDIA@75 QUIZ_QUIZZITCH CUP 2023
 
General knowledge
General knowledge General knowledge
General knowledge
 
3. Penulisan soal HOTS.pdf
3. Penulisan soal HOTS.pdf3. Penulisan soal HOTS.pdf
3. Penulisan soal HOTS.pdf
 
Quiz Competition : MCQ
Quiz Competition : MCQQuiz Competition : MCQ
Quiz Competition : MCQ
 

More from Target PCS Lucknow

Current Affairs Paper Jan 2023 to March 2023 English Solution
Current Affairs Paper Jan 2023 to March 2023 English SolutionCurrent Affairs Paper Jan 2023 to March 2023 English Solution
Current Affairs Paper Jan 2023 to March 2023 English Solution
Target PCS Lucknow
 
Current Affairs Paper Jan 2023 to Mar 2023 Hindi Solution
Current Affairs Paper Jan 2023 to Mar 2023 Hindi SolutionCurrent Affairs Paper Jan 2023 to Mar 2023 Hindi Solution
Current Affairs Paper Jan 2023 to Mar 2023 Hindi Solution
Target PCS Lucknow
 
General Studies Sectional Paper Sample in Hindi - UPPSC Prelims Test Series 2024
General Studies Sectional Paper Sample in Hindi - UPPSC Prelims Test Series 2024General Studies Sectional Paper Sample in Hindi - UPPSC Prelims Test Series 2024
General Studies Sectional Paper Sample in Hindi - UPPSC Prelims Test Series 2024
Target PCS Lucknow
 
UPPSC Current Affairs Practice Paper Hindi Solution
UPPSC Current Affairs Practice Paper Hindi SolutionUPPSC Current Affairs Practice Paper Hindi Solution
UPPSC Current Affairs Practice Paper Hindi Solution
Target PCS Lucknow
 
UPPSC Current Affairs Practice Paper Hindi
UPPSC Current Affairs Practice Paper HindiUPPSC Current Affairs Practice Paper Hindi
UPPSC Current Affairs Practice Paper Hindi
Target PCS Lucknow
 
UPPSC Current Affairs Practice Paper English Solution
UPPSC Current Affairs Practice Paper English SolutionUPPSC Current Affairs Practice Paper English Solution
UPPSC Current Affairs Practice Paper English Solution
Target PCS Lucknow
 
UPPSC Current Affairs Practice Paper English
UPPSC Current Affairs Practice Paper EnglishUPPSC Current Affairs Practice Paper English
UPPSC Current Affairs Practice Paper English
Target PCS Lucknow
 
UPPSC Prelims Test Series Sectional Paper1 Hindi Solution
UPPSC Prelims Test Series Sectional Paper1 Hindi SolutionUPPSC Prelims Test Series Sectional Paper1 Hindi Solution
UPPSC Prelims Test Series Sectional Paper1 Hindi Solution
Target PCS Lucknow
 
UPPSC Prelims Test Series Sectional Paper1 English Solution
UPPSC Prelims Test Series Sectional Paper1 English SolutionUPPSC Prelims Test Series Sectional Paper1 English Solution
UPPSC Prelims Test Series Sectional Paper1 English Solution
Target PCS Lucknow
 
UPPSC Prelims Test Series Sectional Paper1 Hindi
UPPSC Prelims Test Series Sectional Paper1 HindiUPPSC Prelims Test Series Sectional Paper1 Hindi
UPPSC Prelims Test Series Sectional Paper1 Hindi
Target PCS Lucknow
 
Mock paper#1 Model Answer - UPPSC/UPPCS Mains Test Series 2021
Mock paper#1 Model Answer - UPPSC/UPPCS Mains Test Series 2021Mock paper#1 Model Answer - UPPSC/UPPCS Mains Test Series 2021
Mock paper#1 Model Answer - UPPSC/UPPCS Mains Test Series 2021
Target PCS Lucknow
 
Mock paper#1 Question Paper - UPPCS / UPPSC Mains Test Series 2021
Mock paper#1 Question Paper  - UPPCS / UPPSC Mains Test Series 2021Mock paper#1 Question Paper  - UPPCS / UPPSC Mains Test Series 2021
Mock paper#1 Question Paper - UPPCS / UPPSC Mains Test Series 2021
Target PCS Lucknow
 
Mock Paper#1 Model Answer Sample - UPPSC/UPPCS Mains Test Series
Mock Paper#1 Model Answer Sample - UPPSC/UPPCS Mains Test SeriesMock Paper#1 Model Answer Sample - UPPSC/UPPCS Mains Test Series
Mock Paper#1 Model Answer Sample - UPPSC/UPPCS Mains Test Series
Target PCS Lucknow
 
Target PCS Lucknow Sectional Paper#1 Hindi Solution Sample - UPPSC/UPPCS Pre...
Target PCS Lucknow Sectional Paper#1 Hindi Solution Sample -  UPPSC/UPPCS Pre...Target PCS Lucknow Sectional Paper#1 Hindi Solution Sample -  UPPSC/UPPCS Pre...
Target PCS Lucknow Sectional Paper#1 Hindi Solution Sample - UPPSC/UPPCS Pre...
Target PCS Lucknow
 
Target PCS Lucknow Sectional Paper#1 Hindi Sample - UPPSC/UPPCS Prelims Test...
Target PCS Lucknow Sectional Paper#1 Hindi Sample -  UPPSC/UPPCS Prelims Test...Target PCS Lucknow Sectional Paper#1 Hindi Sample -  UPPSC/UPPCS Prelims Test...
Target PCS Lucknow Sectional Paper#1 Hindi Sample - UPPSC/UPPCS Prelims Test...
Target PCS Lucknow
 
Target PCS Lucknow Sectional Paper 1 Hindi Solution Sample - UPPSC/UPPCS Prel...
Target PCS Lucknow Sectional Paper 1 Hindi Solution Sample - UPPSC/UPPCS Prel...Target PCS Lucknow Sectional Paper 1 Hindi Solution Sample - UPPSC/UPPCS Prel...
Target PCS Lucknow Sectional Paper 1 Hindi Solution Sample - UPPSC/UPPCS Prel...
Target PCS Lucknow
 
Sectional Paper 1 Hindi - UPPSC/UPPCS Prelims Test Series Sample 2021 - 2022
Sectional Paper 1 Hindi - UPPSC/UPPCS Prelims Test Series Sample 2021 - 2022Sectional Paper 1 Hindi - UPPSC/UPPCS Prelims Test Series Sample 2021 - 2022
Sectional Paper 1 Hindi - UPPSC/UPPCS Prelims Test Series Sample 2021 - 2022
Target PCS Lucknow
 
UPPSC Pattern Current Affairs May 2020 Hindi - Target PCS
UPPSC Pattern Current Affairs May 2020 Hindi - Target PCSUPPSC Pattern Current Affairs May 2020 Hindi - Target PCS
UPPSC Pattern Current Affairs May 2020 Hindi - Target PCS
Target PCS Lucknow
 
UPPSC Pattern Current Affairs May 2020 english - Target PCS
UPPSC Pattern Current Affairs May 2020 english - Target PCSUPPSC Pattern Current Affairs May 2020 english - Target PCS
UPPSC Pattern Current Affairs May 2020 english - Target PCS
Target PCS Lucknow
 
UPPSC Review Officer (RO/ARO) Test Series full-length paper#1 Hindi - Target ...
UPPSC Review Officer (RO/ARO) Test Series full-length paper#1 Hindi - Target ...UPPSC Review Officer (RO/ARO) Test Series full-length paper#1 Hindi - Target ...
UPPSC Review Officer (RO/ARO) Test Series full-length paper#1 Hindi - Target ...
Target PCS Lucknow
 

More from Target PCS Lucknow (20)

Current Affairs Paper Jan 2023 to March 2023 English Solution
Current Affairs Paper Jan 2023 to March 2023 English SolutionCurrent Affairs Paper Jan 2023 to March 2023 English Solution
Current Affairs Paper Jan 2023 to March 2023 English Solution
 
Current Affairs Paper Jan 2023 to Mar 2023 Hindi Solution
Current Affairs Paper Jan 2023 to Mar 2023 Hindi SolutionCurrent Affairs Paper Jan 2023 to Mar 2023 Hindi Solution
Current Affairs Paper Jan 2023 to Mar 2023 Hindi Solution
 
General Studies Sectional Paper Sample in Hindi - UPPSC Prelims Test Series 2024
General Studies Sectional Paper Sample in Hindi - UPPSC Prelims Test Series 2024General Studies Sectional Paper Sample in Hindi - UPPSC Prelims Test Series 2024
General Studies Sectional Paper Sample in Hindi - UPPSC Prelims Test Series 2024
 
UPPSC Current Affairs Practice Paper Hindi Solution
UPPSC Current Affairs Practice Paper Hindi SolutionUPPSC Current Affairs Practice Paper Hindi Solution
UPPSC Current Affairs Practice Paper Hindi Solution
 
UPPSC Current Affairs Practice Paper Hindi
UPPSC Current Affairs Practice Paper HindiUPPSC Current Affairs Practice Paper Hindi
UPPSC Current Affairs Practice Paper Hindi
 
UPPSC Current Affairs Practice Paper English Solution
UPPSC Current Affairs Practice Paper English SolutionUPPSC Current Affairs Practice Paper English Solution
UPPSC Current Affairs Practice Paper English Solution
 
UPPSC Current Affairs Practice Paper English
UPPSC Current Affairs Practice Paper EnglishUPPSC Current Affairs Practice Paper English
UPPSC Current Affairs Practice Paper English
 
UPPSC Prelims Test Series Sectional Paper1 Hindi Solution
UPPSC Prelims Test Series Sectional Paper1 Hindi SolutionUPPSC Prelims Test Series Sectional Paper1 Hindi Solution
UPPSC Prelims Test Series Sectional Paper1 Hindi Solution
 
UPPSC Prelims Test Series Sectional Paper1 English Solution
UPPSC Prelims Test Series Sectional Paper1 English SolutionUPPSC Prelims Test Series Sectional Paper1 English Solution
UPPSC Prelims Test Series Sectional Paper1 English Solution
 
UPPSC Prelims Test Series Sectional Paper1 Hindi
UPPSC Prelims Test Series Sectional Paper1 HindiUPPSC Prelims Test Series Sectional Paper1 Hindi
UPPSC Prelims Test Series Sectional Paper1 Hindi
 
Mock paper#1 Model Answer - UPPSC/UPPCS Mains Test Series 2021
Mock paper#1 Model Answer - UPPSC/UPPCS Mains Test Series 2021Mock paper#1 Model Answer - UPPSC/UPPCS Mains Test Series 2021
Mock paper#1 Model Answer - UPPSC/UPPCS Mains Test Series 2021
 
Mock paper#1 Question Paper - UPPCS / UPPSC Mains Test Series 2021
Mock paper#1 Question Paper  - UPPCS / UPPSC Mains Test Series 2021Mock paper#1 Question Paper  - UPPCS / UPPSC Mains Test Series 2021
Mock paper#1 Question Paper - UPPCS / UPPSC Mains Test Series 2021
 
Mock Paper#1 Model Answer Sample - UPPSC/UPPCS Mains Test Series
Mock Paper#1 Model Answer Sample - UPPSC/UPPCS Mains Test SeriesMock Paper#1 Model Answer Sample - UPPSC/UPPCS Mains Test Series
Mock Paper#1 Model Answer Sample - UPPSC/UPPCS Mains Test Series
 
Target PCS Lucknow Sectional Paper#1 Hindi Solution Sample - UPPSC/UPPCS Pre...
Target PCS Lucknow Sectional Paper#1 Hindi Solution Sample -  UPPSC/UPPCS Pre...Target PCS Lucknow Sectional Paper#1 Hindi Solution Sample -  UPPSC/UPPCS Pre...
Target PCS Lucknow Sectional Paper#1 Hindi Solution Sample - UPPSC/UPPCS Pre...
 
Target PCS Lucknow Sectional Paper#1 Hindi Sample - UPPSC/UPPCS Prelims Test...
Target PCS Lucknow Sectional Paper#1 Hindi Sample -  UPPSC/UPPCS Prelims Test...Target PCS Lucknow Sectional Paper#1 Hindi Sample -  UPPSC/UPPCS Prelims Test...
Target PCS Lucknow Sectional Paper#1 Hindi Sample - UPPSC/UPPCS Prelims Test...
 
Target PCS Lucknow Sectional Paper 1 Hindi Solution Sample - UPPSC/UPPCS Prel...
Target PCS Lucknow Sectional Paper 1 Hindi Solution Sample - UPPSC/UPPCS Prel...Target PCS Lucknow Sectional Paper 1 Hindi Solution Sample - UPPSC/UPPCS Prel...
Target PCS Lucknow Sectional Paper 1 Hindi Solution Sample - UPPSC/UPPCS Prel...
 
Sectional Paper 1 Hindi - UPPSC/UPPCS Prelims Test Series Sample 2021 - 2022
Sectional Paper 1 Hindi - UPPSC/UPPCS Prelims Test Series Sample 2021 - 2022Sectional Paper 1 Hindi - UPPSC/UPPCS Prelims Test Series Sample 2021 - 2022
Sectional Paper 1 Hindi - UPPSC/UPPCS Prelims Test Series Sample 2021 - 2022
 
UPPSC Pattern Current Affairs May 2020 Hindi - Target PCS
UPPSC Pattern Current Affairs May 2020 Hindi - Target PCSUPPSC Pattern Current Affairs May 2020 Hindi - Target PCS
UPPSC Pattern Current Affairs May 2020 Hindi - Target PCS
 
UPPSC Pattern Current Affairs May 2020 english - Target PCS
UPPSC Pattern Current Affairs May 2020 english - Target PCSUPPSC Pattern Current Affairs May 2020 english - Target PCS
UPPSC Pattern Current Affairs May 2020 english - Target PCS
 
UPPSC Review Officer (RO/ARO) Test Series full-length paper#1 Hindi - Target ...
UPPSC Review Officer (RO/ARO) Test Series full-length paper#1 Hindi - Target ...UPPSC Review Officer (RO/ARO) Test Series full-length paper#1 Hindi - Target ...
UPPSC Review Officer (RO/ARO) Test Series full-length paper#1 Hindi - Target ...
 

Recently uploaded

The Challenger.pdf DNHS Official Publication
The Challenger.pdf DNHS Official PublicationThe Challenger.pdf DNHS Official Publication
The Challenger.pdf DNHS Official Publication
Delapenabediema
 
How to Make a Field invisible in Odoo 17
How to Make a Field invisible in Odoo 17How to Make a Field invisible in Odoo 17
How to Make a Field invisible in Odoo 17
Celine George
 
Model Attribute Check Company Auto Property
Model Attribute  Check Company Auto PropertyModel Attribute  Check Company Auto Property
Model Attribute Check Company Auto Property
Celine George
 
Unit 2- Research Aptitude (UGC NET Paper I).pdf
Unit 2- Research Aptitude (UGC NET Paper I).pdfUnit 2- Research Aptitude (UGC NET Paper I).pdf
Unit 2- Research Aptitude (UGC NET Paper I).pdf
Thiyagu K
 
Multithreading_in_C++ - std::thread, race condition
Multithreading_in_C++ - std::thread, race conditionMultithreading_in_C++ - std::thread, race condition
Multithreading_in_C++ - std::thread, race condition
Mohammed Sikander
 
Synthetic Fiber Construction in lab .pptx
Synthetic Fiber Construction in lab .pptxSynthetic Fiber Construction in lab .pptx
Synthetic Fiber Construction in lab .pptx
Pavel ( NSTU)
 
Chapter 3 - Islamic Banking Products and Services.pptx
Chapter 3 - Islamic Banking Products and Services.pptxChapter 3 - Islamic Banking Products and Services.pptx
Chapter 3 - Islamic Banking Products and Services.pptx
Mohd Adib Abd Muin, Senior Lecturer at Universiti Utara Malaysia
 
Exploiting Artificial Intelligence for Empowering Researchers and Faculty, In...
Exploiting Artificial Intelligence for Empowering Researchers and Faculty, In...Exploiting Artificial Intelligence for Empowering Researchers and Faculty, In...
Exploiting Artificial Intelligence for Empowering Researchers and Faculty, In...
Dr. Vinod Kumar Kanvaria
 
Best Digital Marketing Institute In NOIDA
Best Digital Marketing Institute In NOIDABest Digital Marketing Institute In NOIDA
Best Digital Marketing Institute In NOIDA
deeptiverma2406
 
Overview on Edible Vaccine: Pros & Cons with Mechanism
Overview on Edible Vaccine: Pros & Cons with MechanismOverview on Edible Vaccine: Pros & Cons with Mechanism
Overview on Edible Vaccine: Pros & Cons with Mechanism
DeeptiGupta154
 
Guidance_and_Counselling.pdf B.Ed. 4th Semester
Guidance_and_Counselling.pdf B.Ed. 4th SemesterGuidance_and_Counselling.pdf B.Ed. 4th Semester
Guidance_and_Counselling.pdf B.Ed. 4th Semester
Atul Kumar Singh
 
The basics of sentences session 5pptx.pptx
The basics of sentences session 5pptx.pptxThe basics of sentences session 5pptx.pptx
The basics of sentences session 5pptx.pptx
heathfieldcps1
 
Francesca Gottschalk - How can education support child empowerment.pptx
Francesca Gottschalk - How can education support child empowerment.pptxFrancesca Gottschalk - How can education support child empowerment.pptx
Francesca Gottschalk - How can education support child empowerment.pptx
EduSkills OECD
 
Mule 4.6 & Java 17 Upgrade | MuleSoft Mysore Meetup #46
Mule 4.6 & Java 17 Upgrade | MuleSoft Mysore Meetup #46Mule 4.6 & Java 17 Upgrade | MuleSoft Mysore Meetup #46
Mule 4.6 & Java 17 Upgrade | MuleSoft Mysore Meetup #46
MysoreMuleSoftMeetup
 
Language Across the Curriculm LAC B.Ed.
Language Across the  Curriculm LAC B.Ed.Language Across the  Curriculm LAC B.Ed.
Language Across the Curriculm LAC B.Ed.
Atul Kumar Singh
 
Pride Month Slides 2024 David Douglas School District
Pride Month Slides 2024 David Douglas School DistrictPride Month Slides 2024 David Douglas School District
Pride Month Slides 2024 David Douglas School District
David Douglas School District
 
The Diamond Necklace by Guy De Maupassant.pptx
The Diamond Necklace by Guy De Maupassant.pptxThe Diamond Necklace by Guy De Maupassant.pptx
The Diamond Necklace by Guy De Maupassant.pptx
DhatriParmar
 
Biological Screening of Herbal Drugs in detailed.
Biological Screening of Herbal Drugs in detailed.Biological Screening of Herbal Drugs in detailed.
Biological Screening of Herbal Drugs in detailed.
Ashokrao Mane college of Pharmacy Peth-Vadgaon
 
Chapter -12, Antibiotics (One Page Notes).pdf
Chapter -12, Antibiotics (One Page Notes).pdfChapter -12, Antibiotics (One Page Notes).pdf
Chapter -12, Antibiotics (One Page Notes).pdf
Kartik Tiwari
 
BÀI TẬP BỔ TRỢ TIẾNG ANH GLOBAL SUCCESS LỚP 3 - CẢ NĂM (CÓ FILE NGHE VÀ ĐÁP Á...
BÀI TẬP BỔ TRỢ TIẾNG ANH GLOBAL SUCCESS LỚP 3 - CẢ NĂM (CÓ FILE NGHE VÀ ĐÁP Á...BÀI TẬP BỔ TRỢ TIẾNG ANH GLOBAL SUCCESS LỚP 3 - CẢ NĂM (CÓ FILE NGHE VÀ ĐÁP Á...
BÀI TẬP BỔ TRỢ TIẾNG ANH GLOBAL SUCCESS LỚP 3 - CẢ NĂM (CÓ FILE NGHE VÀ ĐÁP Á...
Nguyen Thanh Tu Collection
 

Recently uploaded (20)

The Challenger.pdf DNHS Official Publication
The Challenger.pdf DNHS Official PublicationThe Challenger.pdf DNHS Official Publication
The Challenger.pdf DNHS Official Publication
 
How to Make a Field invisible in Odoo 17
How to Make a Field invisible in Odoo 17How to Make a Field invisible in Odoo 17
How to Make a Field invisible in Odoo 17
 
Model Attribute Check Company Auto Property
Model Attribute  Check Company Auto PropertyModel Attribute  Check Company Auto Property
Model Attribute Check Company Auto Property
 
Unit 2- Research Aptitude (UGC NET Paper I).pdf
Unit 2- Research Aptitude (UGC NET Paper I).pdfUnit 2- Research Aptitude (UGC NET Paper I).pdf
Unit 2- Research Aptitude (UGC NET Paper I).pdf
 
Multithreading_in_C++ - std::thread, race condition
Multithreading_in_C++ - std::thread, race conditionMultithreading_in_C++ - std::thread, race condition
Multithreading_in_C++ - std::thread, race condition
 
Synthetic Fiber Construction in lab .pptx
Synthetic Fiber Construction in lab .pptxSynthetic Fiber Construction in lab .pptx
Synthetic Fiber Construction in lab .pptx
 
Chapter 3 - Islamic Banking Products and Services.pptx
Chapter 3 - Islamic Banking Products and Services.pptxChapter 3 - Islamic Banking Products and Services.pptx
Chapter 3 - Islamic Banking Products and Services.pptx
 
Exploiting Artificial Intelligence for Empowering Researchers and Faculty, In...
Exploiting Artificial Intelligence for Empowering Researchers and Faculty, In...Exploiting Artificial Intelligence for Empowering Researchers and Faculty, In...
Exploiting Artificial Intelligence for Empowering Researchers and Faculty, In...
 
Best Digital Marketing Institute In NOIDA
Best Digital Marketing Institute In NOIDABest Digital Marketing Institute In NOIDA
Best Digital Marketing Institute In NOIDA
 
Overview on Edible Vaccine: Pros & Cons with Mechanism
Overview on Edible Vaccine: Pros & Cons with MechanismOverview on Edible Vaccine: Pros & Cons with Mechanism
Overview on Edible Vaccine: Pros & Cons with Mechanism
 
Guidance_and_Counselling.pdf B.Ed. 4th Semester
Guidance_and_Counselling.pdf B.Ed. 4th SemesterGuidance_and_Counselling.pdf B.Ed. 4th Semester
Guidance_and_Counselling.pdf B.Ed. 4th Semester
 
The basics of sentences session 5pptx.pptx
The basics of sentences session 5pptx.pptxThe basics of sentences session 5pptx.pptx
The basics of sentences session 5pptx.pptx
 
Francesca Gottschalk - How can education support child empowerment.pptx
Francesca Gottschalk - How can education support child empowerment.pptxFrancesca Gottschalk - How can education support child empowerment.pptx
Francesca Gottschalk - How can education support child empowerment.pptx
 
Mule 4.6 & Java 17 Upgrade | MuleSoft Mysore Meetup #46
Mule 4.6 & Java 17 Upgrade | MuleSoft Mysore Meetup #46Mule 4.6 & Java 17 Upgrade | MuleSoft Mysore Meetup #46
Mule 4.6 & Java 17 Upgrade | MuleSoft Mysore Meetup #46
 
Language Across the Curriculm LAC B.Ed.
Language Across the  Curriculm LAC B.Ed.Language Across the  Curriculm LAC B.Ed.
Language Across the Curriculm LAC B.Ed.
 
Pride Month Slides 2024 David Douglas School District
Pride Month Slides 2024 David Douglas School DistrictPride Month Slides 2024 David Douglas School District
Pride Month Slides 2024 David Douglas School District
 
The Diamond Necklace by Guy De Maupassant.pptx
The Diamond Necklace by Guy De Maupassant.pptxThe Diamond Necklace by Guy De Maupassant.pptx
The Diamond Necklace by Guy De Maupassant.pptx
 
Biological Screening of Herbal Drugs in detailed.
Biological Screening of Herbal Drugs in detailed.Biological Screening of Herbal Drugs in detailed.
Biological Screening of Herbal Drugs in detailed.
 
Chapter -12, Antibiotics (One Page Notes).pdf
Chapter -12, Antibiotics (One Page Notes).pdfChapter -12, Antibiotics (One Page Notes).pdf
Chapter -12, Antibiotics (One Page Notes).pdf
 
BÀI TẬP BỔ TRỢ TIẾNG ANH GLOBAL SUCCESS LỚP 3 - CẢ NĂM (CÓ FILE NGHE VÀ ĐÁP Á...
BÀI TẬP BỔ TRỢ TIẾNG ANH GLOBAL SUCCESS LỚP 3 - CẢ NĂM (CÓ FILE NGHE VÀ ĐÁP Á...BÀI TẬP BỔ TRỢ TIẾNG ANH GLOBAL SUCCESS LỚP 3 - CẢ NĂM (CÓ FILE NGHE VÀ ĐÁP Á...
BÀI TẬP BỔ TRỢ TIẾNG ANH GLOBAL SUCCESS LỚP 3 - CẢ NĂM (CÓ FILE NGHE VÀ ĐÁP Á...
 

UPPSC Review Officer (RO/ARO) Test Series full-length paper#1 English Solution - Target PCS Lucknow

  • 1. RO Full Length Paper#1 Target PCS Lucknow https://targetpcslucknow.com/ Whatsapp/Call @ 7390023092
  • 2. RO Full Length Paper#1 Target PCS Lucknow Page 1 Time Allowed: 2 Hours Maximum Marks: 140 INSTRUCTIONS 1. IMMEDITELY AFTER THE COMMENCEMENT OF THE EXAMINATION, YOU SHOULD CHECK THAT THIS TEST BOOKLET DOES NOT HAVE ANY UNPRINTED OR TORN OR MISSING PAGES OR ITEMS, ETC. IF SO, GET IT REPLACED BY A COMPLETE TEST BOOKLET. 2. You have to enter your Roll Number on the Test Booklet in the Box provided alongside. DO NOT Write anything else on the Test Booklet. 4. This Test Booklet contains 150 items (questions). Each item is printed only in English. Each item comprises four responses (answers). You will select the response which you want to mark on the Answer Sheet. In case you feel that there is more than one correct response, mark the response which you consider the best. In any case, choose ONLY ONE response for each item. 5. You have to mark all your responses ONLY on the separate Answer Sheet provided. See directions in the Answer Sheet. 6. All items carry equal marks. 7. Before you proceed to mark in the Answer Sheet the response to various items in the Test Booklet, you have to fill in some particulars in the Answer Sheet as per instructions sent to you with your Admission Certificate. 8. After you have completed filling in all your responses on the Answer Sheet and the examination has concluded, you should hand over to the Invigilator only the Answer Sheet. You are permitted to take away with you the Test Booklet. 9. Sheets for rough work are appended in the Test Booklet at the end. 10. Penalty for wrong answers: THERE WILL BE PENALTY FOR WRONG ANSWERS MARKED BY A CANDIDATE IN THE OBJECTIVE TYPE QUESTION PAPERS. (i) There are four alternatives for the answer to every question. For each question for which a wrong answer has been given by the candidate, one-third of the marks assigned to that question will be deducted as penalty. (ii) If a candidate gives more than one answer, it will be treated as a wrong answer even if one of the given answers happens to be correct and there will be same penalty as above to that question. (iii) If a question is left blank, i.e., no answer is given by the candidate, there will be no penalty for that question.
  • 3. RO Full Length Paper#1 Target PCS Lucknow Page 2 1.consider the following pillers: Revolutionary Organisation Related Person (1) Anushilan samiti Promotha mitter (2) Ramosi Peasant Force Bhagwan singh (3) Mitra mela V.D.Sawarkar (4) Anjuman-i-Mohisban-i-Watan Ajit Singh Which of the pairs given above are correctly matched ? (a) 1 and 2 only (b) 3 and 4 only (c) 1,3 and 4 only (d)1, 2,3 and 4 only Explanation: C Pairs 1,3 and 4 are correctly matched. Balwant Fadke is related to Ramosi Peasant Force 2. In the context of cultural history of India, the terms ghatikas and matha refers to: (a) traditional pottery works (b) educational institutions (c) poetic literary compositions (d) commercial centers of trade and Commerce Explanation: B Terms ghatikas and matha refers to educational institutions 3.which of the following book were not written by Mahatma Gandhi : 1.Hind swaraj 2. Village Swaraj 3.A study in karma 4.Constructive programme –its meaning and places Explanation: C
  • 4. RO Full Length Paper#1 Target PCS Lucknow Page 3 “A study in karma” is written by Annie Besant 4. The European planters had been forcing the peasant to grow indigo on ____part of the total land 1. 3/10 2. 3/20 3. 3/30 4. 3/50 Explanation: B 5. The theme of Ravana Shaking mount kailash is found at (a) Ajanta Caves (b) Ellora Caves (c) Barabar Caves (d) Bhimbetka Caves Explanation: B The theme of Ravana shaking Mount Kailasha has been found in the caves of Ellora. The image is dated to the eighth century CE. 6. Kiratarjuniyam is a composition of which among the following poets? (a) Kalidas (b) Dandin (c) Bharvi (d) None of the above Explanation: C While some believe that it is the story of the descent of the Ganga from heaven to earth, others believe that the main story is of Kiratarjuniya or Arjuna‘s penance, a poetic work by” Bharvi “ which is known to have been popular in the Pallava court. 7. The term, “sulh-i kul” means: (a) The religion of God (b) Universal peace
  • 5. RO Full Length Paper#1 Target PCS Lucknow Page 4 (c) The rules of war (d) None of the above Explanation: B Sulh-i kul means ―universal peace‖ 8. In Buddhism “Upasaka” means a. Lay Worshipper b. Death of Buddha c. Tantric Buddhism d. First Sermon Explanation: B 9. The Nalayira Divya Prabandham was composed by the: (a) The Alvars (b) The Nayanars (c) The Lingayats (d) None of the above Explanation: B The Nalayira Divya Prabandham is one of the major anthologies of compositions by the Alvars.It was frequently described as the Tamil Veda 10. The Razmnama is a translated version of Indian epic: (a) Mahabharata (b) Ramayana (c) Upanishads (d) None of the above Explanation: A Translations of Sanskrit texts such as the Mahabharata and the Ramayana into Persian were commissioned by the Mughal emperors. The Mahabharata was translated as the Razmnama (Book of Wars) 11. The chief items of export in the Vijayanagar Empire were
  • 6. RO Full Length Paper#1 Target PCS Lucknow Page 5 (a) Horses, pearls and rice (b) Cotton, iron and saltpetre (c) Saltpetre, spices and horse (d) Copper, silk and spices Explanation: B According to the accounts of the foreign travellers, the Vijayanagar Empire was one of the wealthiest parts of the world at that time. The chief items of exports were cotton and silk clothes, spices, rice, iron,saltpetre and sugar. The imports consisted of horses, pearls, copper, coral, mercury, China silk and velvet clothes. 12. Consider the following pairs is not correct Leaders Area of Revolt, 1857 a. General Bakht Khan Bareilly b. Nana Sahib Poona c. Hazrat Mahal Lucknow Explanation: B At Kanpur, the Revolt was led by Nana Sahib, the adopted son of Peshwa Baji Rao II. The most loyal servants of Nana Sahib were Tantia Tope and Azimullah. 13. Tirath Singh is relating to which revolt a. Kandh b. Santhal c. Khasi d. Kol Explanation: C The Khasis, Garos, Khamptis and the Singhpos organised themselves under Tirath Singh to drive away the strangers from the plains. The uprising developed into a popular revolt against the British rule in the area. By 1833, the superior English military force had suppressed the revolt. 14. The Jain Philosophy holds that the world is created and maintained by (a) Universal Soul
  • 7. RO Full Length Paper#1 Target PCS Lucknow Page 6 (b) Universal Truth (c) Universal Faith (d) Universal Law Explanation: D Like the Charvakas, the Jains too do not believe in the Vedas, but they admit the existence of a soul. They believed that the world is created and maintained by Universal Law. 15. The outcome of “Shimla Deputation “was a.Government of india act of 1919 b. Indian council act of 1909 c.Indian council act of 1892 d.Government of india act of 1935 Explanation: B In October 1906, a group of muslim elties called the shimla deputation ,led by the Aga khan , met lord minto and demanded separate electorates for the muslims. with reference to these demands Indian council act of 1909 was passed . 16.Who is called the father of india archeology a.Mortimer wheeler b.John marshall c.James prinsep d.Alexander Cunningham Explanation: D 17. Which of the following statements regarding Tropical cyclone is incorrect ? (a) These are developed only on the hot ocean surface (b) These are formed by different fronts (c) Every parts of tropical cyclone gets rain (d) These are not originated at equator. Explanation: B Temperate cyclone are formed by fronts not tropical cyclone. 18. "Kanto" is the largest plain of which country? (a) Itly (b) Mexico
  • 8. RO Full Length Paper#1 Target PCS Lucknow Page 7 (c) Japan (d) China Explanation: C Kanto is the largest plain of japan . This is the geographical region of the biggest island of japan "Honshu" 19. Sargasso Sea is part of which ocean? (a) Arctic ocean (b) Atlantic ocean (c) Pacific ocean (d) Indian ocean Explanation: B The Sargasso Sea is part of the Atlantic Ocean or the center of marine flora called "Plantak" produced by mixing of hot and cold water streams where fish production is high. 20.The "Kainchee Dam" is built on which river? (a) Niger (b) Nile (c) Zambeji (d) Amazon Explanation: A Kainchee Dam is located on Niger river.Sennar Dam is located on Blue Nile and Kariba Dam is located on Zambezi River. 21.Which of the following areas in the world is not basically dependent on agriculture? (a) The Mediterranean Sea coastal region (b) Egypt's Nile Valley (c) Brahmaputra Valley (d) North-East USA. Explanation: D The North Eastern region of the USA is known as the Developed Industrial Area. Major industrial cities here are Bostow , Portland, Road Iceland Province, Massachusetts etc. Industrial cities which are not famous for agro based industry sector. The rest of the above options basically depend on agriculture. 22.Which of the following gulfs has the world's highest tide? (a) Bay of Bengal (b) Bay of Hudsan (c) Bay of Fundy (d) Bay of Khambhat Explanation: C The highest tide in the world comes in the Bay of Fundy, located on the eastern border of USA and Canada, which is 20% higher than the other Tides. 23.In which state of India is the Subansiri Hydroelectric Project located? (a) Manipur
  • 9. RO Full Length Paper#1 Target PCS Lucknow Page 8 (b) Meghalaya (c) Arunachal pradesh (d) Tamilanadu Explanation: C Subansiri Hydroelectric Project is located on the Subansiri River in Arunachal Pradesh. 24.At what altitude is the alpine vegetation group found above average sea level? (a) 2200-3200 (b) 3200-3700 (c) 3200-4200 (d) 4200-5200 Explanation: B Alpine flora has an average elevation of 32 to 37 meters above sea level. Major trees include silver,fur,juniper, etc. These are economically important. 25.Correct ascending order of creation of following states- (a) Nagaland, Meghalaya, Sikkim, Arunachal Pradesh (b) Meghalaya, Arunachal Pradesh , Nagaland, Sikkim (c) Arunachal pradesh, Nagaland,Sikkim, Meghalaya (d) Sikkim, Nagaland, Arunachal pradesh, Meghalaya Explanation: A Nagaland was formed in 1963, Meghalaya was formed in 1972, Sikkim was formed in 1975 and Arunachal Pradesh was formed in 1987. 26.From which of the following places copper is mined? (a) Kudremukh (b) Koraput (c) Singreni (d) Khetri Explanation: D Copper is produced from Khetri Mining in Rajasthan, while iron ore is produced from Kudremukh and coal is produced from Singareni. 27.Which of the following is a radio wave reflected from atmospheric layers, which is transmitted from the Earth and then back to the Earth? (a) Mesosphere (b) Ionosphere (c) Stratosphere (d) Troposphere Explanation: B Radio waves sent from the Earth to the ionosphere at an altitude of 90 km are reverted so that we can hear or watch broadcasts on our electronic devices. 28.Which of the following statements is not correct with reference to India (a) Iron is abundant in red soil. (b) Phosphorus, nitrogen and organic matter are found in abundance in black soil. (c) Alluvial soil contains a significant amount of potash, but it contains a very small amount of phosphorus.
  • 10. RO Full Length Paper#1 Target PCS Lucknow Page 9 (d) Red soil is suitable for the production of brides and coarse grains. Explanation: B Black soil is formed by the cooling of lava material from volcanic eruptions. This soil is rich in iron, aluminum, magnesium and lime and nitrogen phosphorus and organic matter are not found. Also called regur soil or wheat is world famous for cotton production. 29.Which of the following is not correct? (a) Lake Titicaca -Bolivia - Peru (b) Urmila Lake- Iran (c) Reindeer Lake- Russia (d) Athabasca Lake -Canada. Explanation: C Reindeer Lake is located in Canada. Its depth is 337 meters. Winnipeg Great Bear Great Slave heads in other Canadian lakes. 30.Which of the following statements is not correct? (a) The Boss Strait is located between Australia and Tasmania. (b) The Terrace Strait is located between Australia and New Guinea. (c) The Cook Strait separates the 2 big countries of New Zealand. (d) The Great Australian Bay lies to the east of Australia Explanation: D The Great Australia Bay is an open bay with a coastline in the central western part of Australia. All other statements are true. 31.Which one of the following is not correctly matched? Tribes. Places (a) Kol. Rewa (b) Mudiya. Paudi Garhwal (c) Kodagu. Korba (d) Birhor Ranchi plateau Explanation: B Mudia tribe resides in Bastar district of Chhattisgarh. 32. Rajghat river project jointly established by which two states ? (a) Uttar pradesh and Rajasthan (b) Bihar and jharakhand (c) Madhya pradesh and Maharashtra (d) Uttar pradesh and Madhya Pradesh Explanation: D The Rajghat Dam is a joint project of the state of Uttar Pradesh and Madhya Pradesh located on the Betwa River. 33.Which of the following pairs is not correctly mached? (a) Kudremukh mines- iron ore (b) Talcher - Coal (c) Singareni - Copper (d) Korba-Aluminum. Explanation: C
  • 11. RO Full Length Paper#1 Target PCS Lucknow Page 10 Singrauli is famous for coal production. The remaining statement is true. 34.Which of the following is a source of atmospheric energy? (a) Infrared radiation is emitted downward from clouds. (b) Visible radiation is scattered upward in space. (c) Latent heat is released (d) Albedo of earth and atmosphere. Explanation: C The latent heat present in the water vapor is an indicator of atmospheric energy. It is from these that cyclones get energy. 35.Which of the following processes is important in the creation of electricity in the cloud being formed? (a) Absorption of solar cosmic rays (b) Small lapse rate (c) Intense vertical air stream (d) Heavy rainfall Explanation: C Intense vertical or rising air when condensed above the condensation point creates electricity in a cloud that causes heavy thunderstorms to rain. 36.In which of the following cities was the first City Improvement Trust established? (a) Bombay (b) Calcutta (c) Hyderabad (d) Lucknow Explanation: A The first City Improvement Trust of India was established in Mumbai city. This trust was established keeping in mind the challenges of rapid growth of the Mumbai metropolis and unplanned urbanization. The purpose of this trust was to revisit the mass transit system, planning urban population and strengthening urban institutions. The City Improvement Trust has also been established in other metros of the country under the Mumbai model. 37.Almost all of India's movable oil deposits are located in which of the following rocks? (a) Ancient Granite Deposition Area (b) Ancient Sedimentary Deposition Area (c) Metamorphic Deposition Area (d) Tertiary Depressed Area Explanations: D Almost all of India's movable oil deposits are located in sedimentary rocks dating back to the Tertiary era. Oil deposit refers to mineral oil or petroleum or a mixture of hydrocarbon compounds. The deposit storage of oil in India is in both offshore and onshore areas. The major area of oil deposit in India is as follows. Brahmaputra Valley Gujarat Coast Western Offshore Area, North Eastern Offshore Area. 38.Uttar Pradesh has a number of central universities- (a) 5 (b) 4
  • 12. RO Full Length Paper#1 Target PCS Lucknow Page 11 (c) 3 (d) 2 Explanation: B There are a total of four central universities in Uttar Pradesh. University of Allahabad, Banaras Hindu University, Dr. Bhimrao Ambedkar University, Lucknow and Aligarh Muslim University. 39.Match List 1 with List 2 and choose the correct answer from the code given below the lists - List 1. List 2 Industry. Centre (a) Wooden toy - Meerut (b) Sporting goods. - Bareilly (c) Brass sculptures. - Varanasi (d) Matchmaker industry - Mathura Code:. A. B. C. D (a) 1. 4. 3. 2 (b) 3. 1. 4. 2 (c) 3. 2. 1. 4 (d) 2. 1. 4. 3 Explanation: B Self explanatory 40.Karma folk dance is - (a) Mahoba (b) Poorvanchal (c) Sonbhadra (d) Braj region Explanation: C Karma folk dance is performed by Kharwar tribal people. Its area is Sonbhadra / Mirzapur. 41.Which is the famous pilgrimage place of both Jain and Buddhist religions in Uttar Pradesh? (a) Saranath (b) Kaushambi (c) Kushinagar (d) Shravasti Explanation: B Kaushambi is the famous pilgrimage center for both Jain and Buddhist religions in Uttar Pradesh, while Sarnath and Kushinagar are only related to Buddhism. 42.Renowned Thumri singer Girija Devi is related to - (a) From Jaipur Gharana (b) From Lucknow Gharana (c) From Banaras Gharana. (d) None of the above Explanation: C
  • 13. RO Full Length Paper#1 Target PCS Lucknow Page 12 Renowned Thumri singer Girija Devi is associated with the Banaras Gharana of Uttar Pradesh, a famous music house. 43.Nawabganj Birds Vihar is located- (a) Gonda (b) Unnao (c) Gaziabad (d) Raibarelli Explanation: B Nawabganj Birds santuachury is located in Unnao. It was established in 1984 while Parvati Aranga Bird Sanctuary in Gonda ; Okhla Birds sanctuary in Ghaziabad ; Samaspur Birds santuachry in Rae Bareli . 44. First Biotech park is located in Uttar Pradesh- (a) Lucknow (b) Allahabad (c) Varanasi (d) Ghaziabad Explanation: A Self explanatory 45. Internal Security Academy is located - (a) Mount abu (b) Nasik (c) Pune (d) Hyderabad Explanation: A The Academy of Internal Security is located in Mount Abu, Rajasthan. It was established in 1975 to train officers of the Central Reserve Police Force. 46.Folk dance Rahula is associated with which one of the following regions of UP? (a) From the eastern region (b) From the western region, (c) From the central region (d) From the Bundelkhand region. Explanation: D The folk dance Rawala belongs to the Bundelkhand region of Uttar Pradesh. In this style of laughing laughs, enlightening shlokas sayings, teachings are staged. 47.Rajiv Gandhi Wildlife Conservation Award is given to (a) To Wildlife Conservatives (b) To Forest and Wildlife Officers (c) To Research Institutes. (d) All of above Explanation: D The Rajiv Gandhi Wildlife Conservation Award is presented by the Ministry of Environment and Forests to officers and field workers who have made significant contributions in the field of wildlife conservation research . The award is being given since 1998.
  • 14. RO Full Length Paper#1 Target PCS Lucknow Page 13 48.The main feature of the Act of 1861 was- (a) The Governor General was empowered to issue ordinances when required. (b) The provinces were empowered to enact laws regarding local subjects. (c) Responsive governance was established. (d) All the above statements are true Explanation: A The main feature of the Act of 1861 was. 1.Executive Council of a Governor General was expanded. 2.Departmental system started. 3.The Governor General was given the power to issue ordinances for the first time. 4. The Governor General was empowered to establish the Legislative Council in Bengal, the North Western Frontier Province and Punjab. 49.The President's emergency powers have been adopted under the Indian Constitution (a) From the Constitution of the Soviet Union (b) Government of India Act 1919, (c) From the Weimar Constitution of Germany, (d) From the Constitution of the United States of America. Explanation: C The President of India has the power to suspend fundamental rights during the Emergency in the Indian Constitution. She is very similar to which article of Germany's voimer constitution. Emergency provisions are also derived from the Weimar Constitution of Germany. 50.Which one of the following objectives is not included in the Preamble of the Constitution of India? (a) Freedom of thought (b) Economic freedom (c) Freedom of expression (d) Freedom of belief. Explanation: B The Preamble of the Constitution of India provides for ensuring socio-economic and political justice in addition to providing freedom of thought expression, faith, religion and worship to the citizens. 51.Article 1 of the Indian Constitution declares India as a (a) Union of states. (b) Declares a federal state. (c) Declares a unitary state. (d) Declares a republican state. Explanation: A Article 1 of the Constitution of India declares that India, which is India, will be a union of states, which makes it clear that the Union State is not in any way a result of the mutual agreement of the states because any state should be separated from the Union. Does not have authority. 52.Which one of the following is not correctly matched? State. Article
  • 15. RO Full Length Paper#1 Target PCS Lucknow Page 14 (a) Nagaland. Art.371 A (b) Assam. Art. 371 B (c) Meghalaya. Art. 371 C (d) Andhra pradesh. Art.371 D Explanations: C Art 371 C is related to Manipur. 53. The concept of linguistic state was endorsed (a) Nehru report (b) Dhar commission (c) Cabinet mission (d) Simon commission Explanation: B The commission was constituted by the President of the Constituent Assembly, Dr. Rajendra Prasad. The chairman of this commission was SK Dhar, retired judge of Allahabad High Court. There were 4 members in this commission. The task of this Commission, especially in South India, was to investigate the demand that states should be reorganized on the basis of language or not. This Commission in its report opposed the reorganization of states on the basis of language and on the basis of administrative convenience. But supported the reorganization of states. 54.Which of the following articles in Indian Constitution is about citizenship in India? (a) Art 333 - Art 337 (b) Art 17-Art 20 (c) Art 5 - Art 11 (d) Art 1 - Art 4 Explanation: C There is a provision related to citizenship under Articles 5 to 11 of the Constitution. Articles 1 to 4 have provisions relating to the Union and its territory and Article 17 ends untouchability. Article 18 ends the titles. Article 19 makes provisions for protection of certain rights of speech and expression and Article 20 in relation to conviction for offenses. 55.Which of the statements is correct? (a)The Constitution of India is presidency. (b)India is a nominal monarchy. (c) India is an oligarchy. (d)India is a communal democracy. Explanation:D The expression of the Preamble of the Constitution makes it clear that the Constitution of India has recognized the ultimate source of political power to the people. According to the constitution, the cabinet will be answerable before the house elected by the public and the second part of the executive ie the President is also responsible before the Parliament in the sense that in certain circumstances the Parliament can remove it by impeachment. 56.One point at which equality is found in Indian and American federalism is: (a) the residual powers to be near the center. (b) Residual powers possess states. (c) In some cases there is a provision for concurrent area.
  • 16. RO Full Length Paper#1 Target PCS Lucknow Page 15 (d) In disputes between states, the Supreme Court resolves the dispute. Explanation: D The American federal system and the Indian federal system means that the right has been divided between the union and its units in both countries and the Supreme Court acts as a decider if a dispute arises between the union and its units. 57.Which of the following is stated in the Constitution of India? (a) President will not be a member of any House of Parliament. (b) The Parliament will consist of the President and two Houses. Codes: (a) Only 1 (b) Only 2 (c) 1 and 2 both (d) None of the above Explanation: C Article 79 of the Constitution of India provides that there shall be an MP for the Union, whose President shall consist of the Rajya Sabha and the Lok Sabha. The President shall not be a member of either House on Parliament. 58. If any money bill is approved by the Lok Sabha, then Rajya Sabha can stop it at most- (a) 6 month (b) 4 month (c) 1 month (d) 14 days Explanation: D Article 110 of the constitution states that a bill will be a money bill only if the speaker of the Lok Sabha gives a certificate of its being. The money bill is passed in the Lok Sabha and sent to the Rajya Sabha. If the Rajya Sabha does not take any action within 14 days, it is presumed that it has accepted the bill and will be deemed to have been passed by both the Houses. But if an amendment is passed by the Rajya Sabha, the Money Bill can accept or reject the recommendations of the Lok Sabha. 59.How does the Indian Parliament control the administration? (a) Through Parliamentary committees. (b) Through the consultancy committees of various ministries. (c) Send periodic reports to the administrators. (d) Forcing the executive to issue writ. Explanation:A The Parliament of India controls the administration through committees. For example, it controls public finance through 3 committees. Public Accounts Committee, Estimates Committee, Committee on Public Undertakings. 60.Under which article of Indian constitution can the Governor reserve a bill for the consideration of the President? (a) Art 169 (b) Art 200 (c) Art 201 (d) Art 257
  • 17. RO Full Length Paper#1 Target PCS Lucknow Page 16 Explanation: B Under Article 200, the Governor can reserve any bill passed by the State Legislature for consideration of the President. 61.How long can a member of the Council of Ministers stay in his office without becoming a member of the state assembly? (a) 1 years (b) 3 years (c) 6 month (d) 3 month Explanation: C According to Article 164, no Minister who is not a member of the State Legislature for a continuous period of 6 months shall cease to be a Minister at the end of this period. 62.In which of the following years was the Federal Court of India established? (a) 1935 (b) 1934 (c) 1936 (d) 1937 Explanation: D The Federal Court in India was established in 1933 under the Government of India Act, 1935. 63.Panchayats were granted constitutional status- (a) Art 226 (b) Art 239 (c) Art 243 (d) Art 219 Explanation: C Constitutional status has been accorded to Panchayats under Article 243. A provision has been made in relation to Panchayats under Article 243 of Part 9 of the Constitution. 64.The state government does not have control over local units in any of the following areas (a) Citizen's complaints. (b) Economic matters. (c) Law formation. (d) Personnel matters Explanation:A The 11th schedule has been added to the Constitution by 73 constitutional amendments. The provisions related to Panchayati Raj institutions have been included in this. In this, state governments have been given authority over local bodies over certain areas. Personnel law is important in the economic field, etc., but excludes citizen complaints. 65.The State Election Commission of Uttar Pradesh is - (a) a statutory body. (b) An executive body. (c) A constitutional body. (d) A unit of the Election Commission of India.
  • 18. RO Full Length Paper#1 Target PCS Lucknow Page 17 Explanation: C Article 324 provides for an Election Commission for India. The Election Commission is a constitutional and all-India institution as it is common to both the central and state governments. 66.Which instrument is used to measure height? (a) Ammeter (b) Anemometer (c) Hydrometer (d) Altimeter. Explanation: D Altimeter is a height measuring instrument mainly used in planes. Hydrometer is a comparative density measuring device of water. The ammeter is an electric current measuring device. Anemometer is an air velocity measuring device. 67.What are the atoms that have the same number of protons but different numbers of neutrons? (a) Isobars (b) Isomers (c) Isotherms (d) Isotopes Explanation: D Atoms of the same element with the same atomic number but different mass numbers are called isotopes.The reason why an element has the same atomic number of different isotopes is that they have the same number of protons in their nucleus, but the number of neutrons in them varies, due to which their mass numbers vary. 68.Which of the following is the basic element of carbon? (a) Sand (b) Diamond (c) Marble (d) Suger Explanation: B Diamond is chemically the purest form of carbon. So it is a basic element. The sand is mainly silicon and oxygen. Marble, calcium carbon and oxygen. And sugar is mainly a mixture of carbon, hydrogen and oxygen. 69.Aluminum pages are often anodized. It means depositing on it - (a) Chromium oxide layer (b) Aluminum oxide layer (c) Nickel oxide layer (d) Zinc oxide layer Explanation: B Anodizing is the electrochemical process by which a metal surface is made durable and corrosion resistant. In this process, aluminum oxide layer is deposited on aluminum. 70.Which of the following does not contain carbon? (a)Diamond (b) graphite
  • 19. RO Full Length Paper#1 Target PCS Lucknow Page 18 (c)Coal (d) Sand Explanation: D diamond is the crystalline form of carbon. Its relative density is 3.5. Graphite is a crystalline form of carbon that is a smooth shiny black color while coal is a solid organic material that is used as a fuel. 71.Which of the following gases is used to purify drinking water? (a) Carbondioxide (b) Fluorine (c) Chlorine (d) Helium Explanation: C The Chlorine is used as a disinfectant for making bleaching powder and as a germicide in drinking water. 72.The chemical name of blue thoth is- (a) Sodium bicarbonate (b) Sodium hydroxide (c) Magnesium sulfate (d) Copper sulfate. Explanation: D Blue Thoth is Copper Sulphate ;Epsom Salt is Magnesium Sulphate ;Baking Soda is Sodium Bicarbonate ;Caustic Soda is Sodium hydroxide. 73.Which of the following promotes the ripening of fruits? (a) Carbon dioxide (b) Sulfur dioxide (c) Nitrogen (d) Ethylene Explanation: D Ethylene gas is used in ripening of fruits. 74. What gas is produced in plastic? (a) Polynitrogen (b) Polyhydrones (c) Polychlorin (d) Polyethylene. Explanation: D Polyethylene gas is produced from plastic 75.Which of the following is found in lactic acid, (a) Lemon (b) Butter (c)Milk (d) Vinegar Explanation: C
  • 20. RO Full Length Paper#1 Target PCS Lucknow Page 19 Lactic acid in milk is found in acetic acid in vinegar and citric acid in lemon. Butyric acid is found in butter. 76.Which one of the following is not a Ape? (a) Gibbon (b)Gorilla, (c)Langur (d)Orang Utan Explanation: C The langur is not included in the Ape but in monkey. 77.Which of the following vitamins is called Riboflavin? (a) B1 (b) B2 (c) B6 (d) C Explanation: B The chemical name of B1 is thiamine. Due to its deficiency, there is a disease called Berry Berry. Riboflavin to Vitamin B2. Pyridoxine to vitamin B6. Vitamin C is called ascorbic acid. 78.The National Academy of Agricultural Research Management is located. (a) Bengaluru (b) Hyderabad (c) Kolakata (d) New delhi Explanation: B 79.Which of the following fruits is found in abundant quantity? (a) Jamun (b) Karaunda (c) Lokat (d) Guava. Explanation: B 80.Where is Nauchandi fair held in Uttar Pradesh? (a) Varanasi (b) Pryagraj (c) Meerut (d) Saharanpur Explanation: C 81.According to Indian Council of Agricultural Research, India has number of agricultural ecological zones? (a) 15 (b) 20 (c) 17 (d) 18 Explanation: B
  • 21. RO Full Length Paper#1 Target PCS Lucknow Page 20 82.Total number of census towns in India as per 2011 census- (a) 3894 (b) 4041 (c) 5161 (d) 7935 Explanation: D As per the 2011 population, the total number of census towns in India was 7935 while in 2001 they were 5161. 83.In which city is the Agro Processing Zone located in Uttar Pradesh? (a) Unnao (b) Saharanpur (c) Kanpur (d) Lucknow Explanation: B The Leather Technology Park is in Unnao. Agro Processing Zone is located in Saharanpur, Bio Technology Park in Lucknow, Apparel Park in Kanpur. 84.In Which states locusts enter India? (a) Gujrat (b) Maharastra (c) West Bengal (d) Rajasthan Explanation: D Locuts enters Rajasthan state from Pakistan in India. 85.The Human Development Report is published globally every year: (a) UNDP (b) WTO (c) IMF (d) WORLD BANK Explanation: A UNDP (United Nations Development Programme) 86.Which of the following is not included in the 8 goals of the Climate Action Plan of the Government of India? (a) Solar Power (b) Consumer (c) Nuclear (d) Housing Area. Explanation: C The 8 goals of the Climate Action Plan of the Government of India include the National Solar Mission, National Enhanced Energy Saving Mission, National Sustainable Habitat Mission, National Water Mission, National Himalaya Ecological Preservation Mission, National Green India Mission, National Sustainable Agriculture Mission, National Climate Change Strategic Knowledge Mission. Hence it is clear that it does not include nuclear power.
  • 22. RO Full Length Paper#1 Target PCS Lucknow Page 21 87.In which year was the Water Pollution Prevention and Control Act implemented? (a) 1986 (b) 1981 (c) 1974 (d) 1972 Explanation: C The Central Pollution Control Board of India was formed in September 1974 under the Water Pollution Prevention and Control Act 1974 as a statutory organization. It provides technical services to the Ministry of Environment and Forests under the provisions of the Environment Protection Act 1986. 88.Which of the following is not a water borne disease? (a) Typhoid (b) Hepatitis (c) Cholera (d) Dengue Explanation: D Typhoid, hepatitis is a waterborne disease. Dengue is caused by a type of virus infection that is spread by a fly called Aedes aegypti. 89.Sort the following National Parks of India from north to south direction: (a) Indravati National Park, (b) Nagarhole National Park (c) Corbett National Park (d) Madhav National Park. Codes: (a) 2,3,4,1 (b) 2,1,4,3 (c) 1,3,4,2 (d) 3,4,1,2 Explanation: D Corbett National Park, Uttarakhand; Madhav National Park, Madhya Pradesh; Indravati National Park, Chhattisgarh; Nagarhole National Park, Karnataka. 90.Which one of the following is a source of methane emission in the atmosphere? (a) Automatic vehicle vacuum sputter (b) Industrial chimney (c) Mining (d) Wetlands. Explanation: D Methane gas emissions predominantly from leaks in wetlands such as rice fields, livestock, land, and natural gas systems or account for 11% of greenhouse emissions. 91.In geographical areas, Kanha National Park is related to which one? (a) Tropical low wet forest (b) Tropical wet forest (c) Tropical dry forest (d) Subtropical forest
  • 23. RO Full Length Paper#1 Target PCS Lucknow Page 22 Explanation: A The biographical areas of Kanha National Park are related to tropical low humid forest 92.The range of the number of animals in a certain area that the environment can support is called the (a)Population. (b) Carrying capacity (c)Number or the pyramid of bio mass (d)None of the above. Explanation: B 93.Nitrogen oxide is the major source of pollution in urban areas (a) Energy power plants (b) Road transport (c) Commerce sector (d) Industries. Explanation: B Nitrogen oxide originates from the burning of mineral oil and coal. Due to excessive concentration of nitric oxide in the human body and many diseases it suffers from diseases. Such as gingivitis, bleeding, pneumonia, lung cancer 94.When is Biodiversity Day celebrated? (a) 28 Feburary (b) 29 December (c) 27 June (d) 22 May Explanation: B Biodiversity Day is celebrated by United Nations on 22 May, while many countries of the world celebrate Biodiversity Day on 29 December. It is noteworthy that the Convention on Biodiversity was effective from 29 December 1993. 95. YASH Program, recently in the news, is related to: (a) Gold monetisation (b) Health and risk communication (c) Employment for rural workers amid Covid-19 outbreak (d) Welfare measures for defence personnel Explanation: B The National Council for Science & Technology Communication (NCSTC), Department of Science & Technology (DST) has launched a programme on health and risk communication ‘Year of Awareness on Science & Health (YASH)’ with focus on Covid- 19.The programme is a comprehensive and effective science and health communication effort for promoting grass-root level appreciation and response on health. 96.When did the World Trade Organisation come into effect?
  • 24. RO Full Length Paper#1 Target PCS Lucknow Page 23 (A) March 6, 1996 (B) April 8, 1994 (C) February 5, 1994 (D) January 1, 1995 Explanation: D The World Trade Organization (WTO) is an intergovernmental organization that is concerned with the regulation of international trade between nations. The WTO officially commenced on 1 January 1995 under the Marrakesh Agreement, signed by 123 nations on 15 April 1994, replacing the General Agreement on Tariffs and Trade (GATT), which commenced in 1948. It is the largest international economic organization in the world. 97.Mount Harriet National Park, recently in the news, is located in: (a) Karnataka (b) Dada and Nagar Haveli (c) Goa (d) Andaman and Nicobar Islands Explanation: D Mount Harriet National Park is a national park located in the Andaman and Nicobar Islands union territory of India. The park, established in 1969. 98.Which Indian state/UT has formed a dedicated commission to create employment opportunities for the migrant labourers? (a) Odisha (b)Madhya Pradesh (c)Uttar Pradesh (d)Bihar Explanation: B The state government of Madhya Pradesh has formed a dedicated commission to create employment opportunities for the migrant labourers. Named as the Madhya Pradesh State Migrant Labourer Commission (MPSMLC), it aims to create jobs to those who have returned from other parts of the country during the Covid 19- induced lockdown. 99.What is the theme of the ‘International Day of Parliamentarism’ 2020? (a)Parliaments in a time of pandemic (b) Role of Parliaments (c)Democracy and Parliament (d) Parliaments Matter Explanation: A [Parliaments in a time of pandemic] June 30 is the day designated by the United Nations to celebrate the International Day of Parliamentarism.
  • 25. RO Full Length Paper#1 Target PCS Lucknow Page 24 The United Nations General Assembly recognized the role of parliaments in developing the world, and designated the special day. On the same day in the year 1889, the global organization of parliaments called the Inter-Parliamentary Union (IPU) was established. 100.The term “SPICe”, sometimes seen in news, is related to which of the following fields? (a) Science & Technology (b)Environment Protection (c) Business & Economy (d) Defence and Arms Production Explanation: C Ministry of Corporate Affairs is replacing the existing SPICe (Simplified Proforma for Incorporating Company Electronically) form with a new web form called SPICe+. 101.Who is the head of the High level expert group on farm exports, recently set up by the 15th Finance Commission? (a) N K Singh (b) U K Sinha (c) Tapan Ray (d) Sanjiv Puri Explanation: D The 15th Finance Commission (FFC) recently set up a High level expert group on farm exports. The group is to be headed by the Chairman and Managing director of ITC, Sanjiv Puri.The committee will suggest performance-based incentives to the state governments for 2021-22 to 2025-26, to accelerate agricultural reforms and exports in the sector. 102.In which year Government of India started Navratna category? (a)1992 (b) 1993 (c) 1995 (d) 1997 Explanation: D (1997) The Public Sector Enterprises are run by the Government under the Department of Public Enterprises of Ministry of Heavy Industries and Public Enterprises. The government grants the status of Navratna, Miniratna and Maharatna to Central Public Sector Enterprises based upon the profit made by these CPSEs. The Maharatna category has been the most recent one since 2009, other two have been in function since 1997. 103.The portion of profits that a company distributes among its shareholders in the form of cash is usually known as _____? (a) Yield (b) Dividend (c) Stock Split (d) Free Float Explanation: B Dividend is the portion of profits that a company distributes among its shareholders in the form of cash. Usually it is expressed per share. In some cases it is expressed as a percentage of the share`s face value.
  • 26. RO Full Length Paper#1 Target PCS Lucknow Page 25 104.What is the number of indicators in Multi-dimensional Poverty Index (MPI) ? (a) 8 (B)9 (c) 10 (d) 11 Explanation: C Dimension Indicators Deprivation Cutoffs [10][11] Health Child mortality Deprived if a child under the age of 18 years has died in the family In the 5 years preceding the survey. Nutrition Deprived if any adult or child, for whom there is nutritional information, is undernourished. Education Years of schooling Deprived if no household member has completed six years of schooling. School attendance No household member aged 'school entrance age + six' years or older has completed six years of schooling. Standard of living Cooking fuel Deprived if the household cooks with dung, wood or charcoal. Sanitation Deprived if the household's sanitation facility is not improved (according to MDG guidelines), or it is improved but shared with other households. Drinking water Deprived if the household does not have access to improved drinking water (according to MDG guidelines) or improved drinking water is more than a 30-minute walk from home round trip. Electricity Deprived if the household has no electricity. Housing Deprived if at least one of the three housing materials for roof, walls and floor are inadequate: the floor is of natural materials and/or the roof and/or walls are of natural or rudimentary
  • 27. RO Full Length Paper#1 Target PCS Lucknow Page 26 materials. Assets Deprived if the household does not own more than one of these assets: radio, TV, telephone, computer, animal cart, bicycle, motorbike or refrigerator and does not own a car or truck 105.In India, Banks can accept interest free deposits in ___ (a)Current Account (b)Saving Account (c)Demand Deposits (d)Term Deposits Explanation: A Banks cannot accept interest free deposits other than in current account. 106.Which of the following is / are auctioned in Open Market Operations? (a) Shares (b) Debentures (c) Securities (d) Bullion Explanation: C Open Market Operations refer to the purchase and sale of the Government securities (G- Secs) by RBI from / to market. 107.Which Investment bank collapse started the 2008 global financial crisis? (a) Morgan Stanley (b)Goldman Sachs (c)Lehman Brothers (d) Merill Lynch Explanation: C The collapse of Lehman Brothers in 2008 has started the crisis in 2008. The bank filed bankruptcy after it failed to repay its payments. It was an investment banking firm. 108.Sahyog-Kaijin’ is an annual joint exercise between the Coast Guards of India and which country? (a) Bangladesh (b) China (c) Japan (d) Sri Lanka Explanation:C Sahyog-Kaijin’ is an annual joint exercise between the Coast Guards of India and Japan. The 19th edition of the exercise is scheduled to begin in Chennai, India.
  • 28. RO Full Length Paper#1 Target PCS Lucknow Page 27 109.What is the theme of the DefExpo 2020, the 11th edition of the biennial defence exhibition? (a)India: The Emerging Defence Manufacturing Hub (b)Artificial Intelligence in Defence (c) Make in India- A pride in Defence (d) Digital battlefield of the future Explanation: A The 11th edition of the biennial mega defence exhibition, DefExpo 2020 is to be held in the capital city of Uttar Pradesh- Lucknow from February 5 to 9, 2020. The theme of this edition of the event is ‘India: The Emerging Defence Manufacturing Hub’. The sub-theme is announced as ‘Digital Transformation of Defence’. 110.According to the recent Economic Survey 2019-20, which state has the cheapest Vegetarian Thali (food plate)? (a) Kerala (b) Jharkhand (c) Bihar (d) Tripura Explanation: B The term called ‘Thalinomics’ was used, which means the affordability of buying a Thali (Food Plate). It states that, Jharkhand has the cheapest Vegetarian Thali (meals plate), less than Rs.50. 111.Which Indian businessperson has entered the top-10 of Forbes India Rich List 2020 for the first time? (a) Gautam Adani (b) Shiv Nadar (c)Cyrus Poonawalla (d) Uday Kotak Explanation: C As per the Forbes India Rich List 2020 that was released recently, the chairman of Reliance Industries Limited Mukesh Ambani remains the richest Indian for the 13th consecutive year.His net worth stands at USD 88.7 billion.The new entrant in the top 10 is Cyrus Poonawalla, the Chairman of Serum Institute of India (SII). 112.Who has been recently awarded with the Subash Chandra Bose Aapda Prabandhan Puraskar, 2020? (a) Vijay Kumar (b) Sylendra Babu (c) Kumar Munnan Singh (d) Naveen Kumar Explanation: C Kumar Munnan Singh has been selected for the Subhash Chandra Bose Aapda Prabandhan Puraskar 2020 under the Individual category. Under the Institution category, Disaster Mitigation & Management Centre, Uttarakhand has been selected for the award. Kumar
  • 29. RO Full Length Paper#1 Target PCS Lucknow Page 28 Munnan Singh was the founding member of the National Disaster Management Authority in 2005. He is also lauded for establishing the National Disaster Response Force (NDRF). 113.Which Indian sportsperson is set to take charge as the President of the Paralympic Committee of India? (a) Devendra Jhajharia (b) Deepa Malik (c) Mariyappan Thangavelu (d) Varun Singh Bhati Explanation: B india’s first and only woman Paralympics silver medallist, Deepa Malik is set to take charge as the President of the Paralympic Committee of India. 114.‘World Consumer Rights Day’, 2020 is - (a) 12 March (b) 13 March (c) 14 March (d) 15 March Explanation: D World Consumer Rights Day is celebrated every year on March 15 across the world to spread awareness about the rights and needs of a consumer. Theme of this year is “Sustainable Consumer”. 115.When did this year’s Earth Overshoot Day fall? (a) August 20 (b) August 22 (c) June 19 (d) June 22 Explanation: B This year, the Earth Overshoot Day fell on 22nd August, 2020. Earth Overshoot Day is the date when the demand for ecological resources and services by human beings exceeds what Earth can regenerate in a given year.This is computed by the think tank named Global Footprint Network. 116.Axone, a popular condiment of Nagaland is made from which of the following? (a) Rice (b) Wheat (c) Corn (d) Soya bean Explanation: D 117.Global Economic Prospects report is released by? (a) World Bank (b) World Trade Organisation (c) International Monetary Fund (d) World Economic Forum Explanation:A
  • 30. RO Full Length Paper#1 Target PCS Lucknow Page 29 118.Which numbers will come next in the following series? 21 11 6 3.5 ? (a) 2.5 (b)1.5 (c)2 (d)2.25 Explain- D 21+1 and divided by 2=11 11+1 and divided by 2=6 So 3.5+1 and divided by 2 = 2.25 119. Choose the correct option.. 54:9::?:7 (a) 63 (b) 42 (c) 21 (d) 28 Explain (B) 9×6=54 so 7×6=42 120. Which one of the following number is different from other? (a) 96 (b) 87 (c) 69 (d) 68 Explain (D) Because sum of the number is 15 121. Which one of the following is different from other three?
  • 31. RO Full Length Paper#1 Target PCS Lucknow Page 30 (a) Shyam: honest (b) Ram: loyal (c) Sita:lazy (d) Sunder:dark face Explain ( D) dark face is god gifted 122. In a certain code if RAM = 321 NAVI = 6245 HARIOM = 92381 MOHAN = ?? (a) 54321 (b) 19876 (c) 18926 (d) 89678 Explain (C ) number used as word code 123. As Sita is related to Ram then in which of them related to shyam?? (a) Dwarika (b) Radha (c) Sudama (d) Gwal Explain (B) used as god
  • 32. RO Full Length Paper#1 Target PCS Lucknow Page 31 124. Which one of the following is different from other three? (a) HI = 17 (b) MN = 13.5 (c) PQ = 16.5 (d) ZS = 22.5 Explain (A) sum of the place value of the word and divided by 2 125. Which one of them is not created from 'INFORMATION'? (a) FROM (b) FOR (c) MENTION (d) MOTION Explain (C) E is not in INFORMATION 126. Fill the blanks.. COMP_ _ _ _ (a) OTER (b) UTER (c) UTAR (d) UTRE Explain (B) COMPUTER
  • 33. RO Full Length Paper#1 Target PCS Lucknow Page 32 127. Which one of the following is different from other three? (a) 9×11=99 (b) 8×9=72 (c) 14×12=84 (d) 16×14=64 Explaination :(D) LCM of the number 128.CogX, a global leadership summit, is related to: (a) Artificial Intelligence & Emerging Technology (b) Preservation of Corals (c) Space Junk (d) Freedom of Press Explanation: A CogX is a prestigious Global Leadership Summit and Festival of Artificial intelligence & Emerging Technology held annually in London with over 15,000 participants in attendance from the highest levels of business, government, industry, and research. 129.The ‘Tangam language’ which is marked ‘critically endangered’ by UNESCO is spoken in which state? (a) Kerala (b) Chattisgarh (c) Arunachal Pradesh (d) Mizoram Explanation: C tangam is a little-known community within the larger Adi tribe of Arunanchal pradesh which resides in the hamlet of Kugging in Upper Siang district’s Paindem circle.For long, the only account of the Tangams could be found in a book, “Tangams” (1975) where the community’s population was pegged at 2,000 spread across 25 villages. 130. In which year The Central Drug Reaserch Institute was established (a) 1949 (b) 1950 (c) 1951 (d) 1952 Explanation: C The Central Drug Research Institute was one of the first laboratories to be established in India right after its independence. It is among the thirty nine laboratories that are functioning under the aegis of the council of scientific and Industrial Research Council of Scientific and Industrial Research of India. The research institute was formally inaugurated on 17 February 1951 by Prime Minister, Jawahar Lal Nehru.
  • 34. RO Full Length Paper#1 Target PCS Lucknow Page 33 131.. Recently in news, the ‘Peace Forest Initiative’ has been launched by: (a) South Korea (b) India (c) Brazil (d) Bhutan Explanation: A Peace Forest Initiative: It is an initiative of South Korea to use ecological restoration as a peace-building process. It aims at addressing the issue of land degradation in conflict-torn border areas and would go a long way in alleviating tensions and building trust between communities living there and between enemy countries in particular. 132.Which of the following water bodies borders Saudi Arabia: 1. Red Sea 2. Gulf of Aden 3. Persian Gulf 4. Mediterranean Sea Choose the correct option: (a) 1 and 3 only (b) 2 and 4 only (c) 2, 3 and 4 only (d) 1, 3 and 4 only Explanation: A 133.The first census was conducted in India - (a) 1861 (b) 1972 (c) 1883 (d) 1881 Explanation: D A systematic and modern population census, in its present form was conducted non synchronously between 1865 and 1872 in different parts of the country. This effort culminating in 1872 has been popularly labeled as the first population census of India However, the first synchronous census in India was held in 1881. Since then, censuses have been undertaken uninterruptedly once every ten year. 134.Danakil Depression is located in which of the following country? (a) Australia (b) Yemen (c) Ethiopia (d) Russia Explanation: C The Danakil Depression is the northern part of the Afar Triangle or Afar Depression in Ethiopia, a geological depression that has resulted from the divergence of three tectonic plates in the Horn of Africa. 135.What is the correct descending order of Rice producing states in india? (a) Punjab, West Bengal, Uttar Pradesh And Andhra Pradesh (b) Punjab, West Bengal, Uttar Pradesh And Andhra Pradesh (c) Punjab, West Bengal, Uttar Pradesh And Andhra Pradesh (d) West Bengal, Uttar Pradesh, Punjab, And Andhra Pradesh
  • 35. RO Full Length Paper#1 Target PCS Lucknow Page 34 Explanation: D Top Rice Producing States: West Bengal > Punjab > Uttar Pradesh > Andhra Pradesh > Bihar. It is the staple food crop of majority of Indian people. India is the second largest producer of rice in the world after China. In states like Assam, West Bengal and Odisha, three crops of paddy are grown in a year. 136.Who approves the Fair and Remunerative Price (FRP) of sugarcane? (a) Cabinet Committee on Economic Affairs (b) Commission for Agricultural Costs and Prices (c) Directorate of Marketing and Inspection, Ministry of Agriculture (d) Agricultural Produce Market Committee Explanation: A The Federal/Central Government announces Fair and Remunerative Prices which are determined on the recommendation of the Commission for Agricultural Costs and Prices (CACP) and are announced by the Cabinet Committee on Economic Affairs, which is chaired by Prime Minister.The State Advised Prices (SAP) are announced by key sugarcane producing states which are generally higher than FRP. 137.The persons who became the Chief Minister for the longest period in India ? (a) E.K. Nayanar (b) Vasantha Sate (c) Jyothi Basu (d) K. Karunakaran Explanation: C Jyoti Basu was an Indian politician and statesman who served as the Chief Minister of West Bengal state from 1977 to 2000 138. In which place the first Medical college was established in India ? (a) Kolkota (b) Agra (c) Bangalore (d) Delhi Explanation: A Calcutta Medical College, officially Medical College and Hospital, Kolkata, is an Indian medical school and hospital. The school was established in 1835 by Lord William Bentinck as Medical College, Bengal during British Raj.
  • 36. RO Full Length Paper#1 Target PCS Lucknow Page 35 139.Who introduced English education in India ? (a) Cornwalis (b) Rippon (c) Delhousie (d) William Bentinck Explanation: D 140.Van Vihar National Park, recently seen in news, is located in (a) Andhra Pradesh (b) Meghalaya (c) Odisha (d) Madhya Pradesh Explanation: D Van Vihar National Park is a national park in Bhopal, the capital city of Madhya Pradesh in central India. Declared a national park in 1979